Sie sind auf Seite 1von 80

Mitral stenosis, what murmur

Low pitched diastolic murmur at the apex with the bell, usually with a diastolic
rumble. Opening snap after S2 as well.
Can lead to atrial fibrillation due to left atrial enlargement. As well as
pulmonary hypertension --> hemoptysis and signs of right heart failure such as
peripheral edema. + AF --> RVR produces pulmonary congestion

AFib different forms?

Acute
Paroxysmal
Chronic

ECG findings in AFib

Absence of discrete P waves


Irregularly irregular ventricular
contraction

Major causes of morbidity with AFib

RVR --> ischemia or exacerbation of heart


failure
Thrombus with embolization

Two most common causes of AFib

Hemodynamically stable AFib patients

Most effective method of terminating AF

Hypertension
Coronary atherosclerosis
Other causes:
Structural heart disease (HTN, mitral valve disease)
Ischemic heart disease
Pericarditis or pericardial injury (post surgical)
Pulm disease (esp PE)
Hyperthyroidism
Stress or increased sympathetic tone (acute illness, pheochromocytoma)
Alcohol consumption ("holiday heart syndrome," alcoholic cardiomyopathy)
Sick sinus syndrome (tachy brady syndrome

Ventricular rate control --> IV BBs, CCBs,


digoxin (which slows conduction through
AV node).
Electrical cardioversion
>48 hours? Either 3-4 weeks of warfarin, or in low risk patients, TEE. Postcardioversion anticoagulation is still
required for 4 weeks.

Also, pharm agents such as propafenon, sotalol, and amiodarone may be tried to maintain sinus rhythm.

Important prognostic factors for


maintenance of sinus rhythm after AF
cardioversion
CHA2DS2 VaSc score

Left atrial dilation (diameter >4.5cm)


Duration of AF

Congestion heart failure


Htn
Age >75 -2 popints
Diabetes
Stroke - 2 points
Vascular disease
Sex --> females - 1 point

Risk of stroke with chronic AFib

1-5% risk per year...


+ valvular disease, it is way higher.
Anticoagulaion with warfarin decreases risk of stroke
by 2/3.

INR with warfarin, when would you give


Vitamin K

Most frequently involved valve in


rheumatic heart disease

ECG findings in WPW

INR>9
If significantly increased bleeding is present, vitamin
K and FFP, intravascular volume replacement.
Mitral valve stenosis
Aortic valve too, but usually in combo with mitral valve
Right side of heart is rarely involved.

Delta wave
Widening of the QRS
Shortening of the PR interval
Some may be asymptomatic, others may develop recurrent tachyarrhythmias. Most by PVST....
others by AFib, which may look like a ventricular tachycardia.... but it will have the irregular RR
interval of AF.

AF in the setting of WPW... treatment?

You do NOT do the normal AV nodal blocking drugs because the impulse is not traveling through
the AV node... BB, CCB can actually paradoxically increase Ventricular rate and should be
avoided...
DC cardioversion can be treated if unstable
Stable? Procainamide, or Amiodarone, to slow condution and convert rhythm to sinus.

Diastolic rumble at the apex heard with


the bell

Mitral stenosis

Early diastolic decrescendo at the RUSB

Aortic regurg

Holosystolic murmur at the apex

Mitral regurg

Late peaking systolic murmur at the RUSB

Aortic stenosis

Most common causes of AFib

HTN
Atherosclerotic heart diseaes
Pericardial or pulmonary disease
Hyperthyroidism

How do you decide "lone atrial


fibrillation"?

CHADS2 score <2

AFIB + WPW

Unstable - DC cardioversion
Stable - Procainamide or Amiodarone

Interval between S2 and opening snap of


mitral stenosis varies with what?

Inversely varies with the severity of the


stenosis.

Findings suggestive of a duodenal ulcer

Pain is burnign
Occurs when the stomach is empty
Relieved by food or antacids

Dyspepsia

Pain or discomfort centered in the upper abdomen (mainly in or around the


midline), which can be associated with fullness, early satiety, bloating, nausea.
Can be intermittent or continuous, and it may or may not be related to meals.
Can be functional! In which cause persisting for 12 weeks but without evidence
of ulcer on endoscopy.

H. pylori and relation to stomach and


duodenal ulcers

Stomach - 50-60% associated


Duodenal - 70-90% associated

H. pylori, what type of organism

Gram negative microaerophilic bacillus

GERD vs biliary colic vs PUD

GERD - burning epigastric or mid chest pain usually occurring after meals and worsening with recumbency.
Biliary colic - acute onset of severe pain in RUQ or epigastrium, precipitated by meals, esp fatty foods, lasts 30-60 minutes
with spontaneous resolution
Gastric ulcers - Food may worsen symptoms, some may have no symptoms at all... 5-10% are malignant
Duodenal ulcers - Pain worse after stomach is emptied (but food stimulated acid production persists), typically 2-5 hours
after a meal. Food may relieve pain.

Gastric cancers, how may they present

Dysphagia if in the cardia


Persistent vomiting if they block pyloric channel
Early satiety by their mass effect or infiltration of hte
stomach wall.

Who gets endoscoped looking for gastric


cancers

New onset dysphagia


Older than 45 yo.
Alarm symptoms: weight loss, recurrent vomiting, dysphagia, evidence
of GI bleeding, Fe deficiency anemia.
Those who have failed to respond to empiric therapy for PUD./GERD

Young patients with no alarm features


wtih dyspepsia

Serology for H. pylori (will remain positive for life)


Urea breath test (active infection)
Fecal H pylori antigen test

Regimens for PUD caused by H. pylori

Omeprazole OR Bismuth subsalicylate


Clarithromycin
Metronidazole OR Amoxicillin
Whether treatment of H pylori infection reduces or eliminates dyspeptic sx in
the absence of ulcers (nonulcer dyspepsia) is uncertain

Complications of H pylori infection beyond


ulcers

How do NSAIDs cause ulcers?

Gastric carcinoma
Gastric mucosa-associated lymphoid tissue
(MALT) lymphoma
Inhibit prostaglandin synthesis, which results in reduced secretion of mucus
and bicarb, and decreased mucosal blood flow.
This is dose dependent, and can occur within days after treatment is initiated.
If ulceration occurs, NSAID should be discontinued if possible. Acid
suppression therapy should be started.

When should you suspect Zollinger Ellison


syndrome

Who gets gastrinomas?

Ulcers refractory to treatment


Peptic ulceration
Often diarrhea.
ulcers in unusual location (beyond duodenal bulb), or ulcers
wtihotu a history of NSAID use or H. pylori infection...
25% occur in the context of MEN1, an AD disorder characterized by
parathyroid., pancreatic, and pituitary neoplasms.
MEN1 - Wermer's sydnrome

MEN1 vs MEN2A vs MEN2B

MEN1 - Parathyroid, Pituitary tumors (prolactin or GH), Pancreatic endocrine tumors (ZE, insulinomas,
VIPomas, glucagonomas (rare)). Commonly presents with kidney stones and stomach ulcers
MEN2A - Medullary thyroid carcinoma (secretes calcitonin), Pheo, Parathyroid
MEN2B - Medullary thyroid carcinoa, Pheo, Oral/intestinal ganglioneuromatosis (assoc with marfanoid
habitus).

Multiple endocrine neoplasia assoc with


marfanoid habitus

MEN2B: Medullarythyroid, pheo, and


oral/intestinal ganglioneuromatosis

How to diagnose Zollinger Ellison?

Measure serum gastrin, which may be


>1000pg/ml
Localization of tumor with imaging studies.

H. pylori is associated with what

Antibiotics good for H. pylori

Duodenal and gastric ulcers


Chronic active gastritis
Gastric adenocarcinoma
Gastric mucosa-associated lymphoid tissue lymphoma

Clarithromycin
Amoxicillin
Metronidazole
Tetracycline

Osmotic cerebral demyelination is AKA

Central pontine myelinolysis.


Osmotic cerebral demyelination is preferred because
the demyelination can occur anywhere in the brain.

Goal for treatment of symptomatic


hyponatremia

Level of safety to 120-125 or so.


Hyponatremia definition is <135

Hyponatremia, symptoms?

Asymptomatic until low 120s, in which there is


enough cerebral edema to cause headache, nausea,
vomiting, with later symptoms --> lethargy,
confusion, seizures, or coma.

Hypotonic hyponatremia is ALWAYS an


indicator of what

Water gain--> restriction or impairment of


free water secretion.

Normal kidney capacity for free water


secretion

18-20 L/day, therefore polydipsia is hard to


do.

Hypovolemic hypoosmolar hyponatremia?

Correction of volume status usually with


isotonic saline.

Hypervolemic hypoosmolar hyponatremia


causes

CHF
Cirrhosis
Nephrotic syndrome
Administration of diuretics....

Euvolemic hypoantremia with urine that is


not as dilute as it should be, consider what
common diagnoses?
How to diagnose hyponatremia due to
SIADH

Hypothryoidism
Adrenal insufficiency -- coritsol deficiency
Thyroid hormone and cortisol are both permissive for free water excretion, so their deficiency
causes water retention
But it is most commonly due to SIADH, but this is a diagnosis of exclusion!

Euvolemic hypoosmolar hyponatremia with UNa>20 or UOsm


>150-200, urine sodium more than 20 mmol/l, and normal adrenal
and thyroid function.
Some other lab clues: low BUN and low uric acid levels...

How might severe hyponatremia that is


clinically evident but euvolemic be
treated?

Hypertonic saline (3%) with a loop diuretic


such as furosemide... This will cause
excretion of hypotonic urin...

General rule for sodium correction

Chronic hyponatremia --> no faster than


0.5-1 mEq/hr

Drugs for possible treatment of chronic


hypervolemic hyponatremia?

As in heart failure or cirrhosis --> vasopressin antagonists such as


vaptans are available and are very effective in increasing free water
excretion and raising serum sodium concentrations. This is
typically done in a hospital setting.

Postop period or pain may trigger


hyponatremia, how?

RElease of vasopressin (ADH) leading to the


inappropriate retention of free water, which leads to
dilution of hte serum. Concomitant administration of
hypotonic fluids may exacerbate the situation.

Low Na
High K
Acidosis

Adrenal insufficiency
Also have fatigue, weight loss, low BP, and
hyperpigmentation...

How to make diagnosis of Addison's


disease?

24 h urine cortisol test


Measuring cortisol in response to ACTH,
showing low cortisol levels...

Conn syndrome

Hyperaldosteronism --> hypertension and


hypokalemia and an alkalosis

Initial treatment of hypovolemic patients


with hyponatremia

Volume replacement typically with


isotonic (0.9%) saline

Treatment of euvolemic asymptomatic


hyponatremia

Fluid restriction.

Rate of sodium correction in hyponatremia

Should not exceed 0.5-1mEq/h

Aortic dissection features

BP discordance between right and left arms (L>R)


Soft early diastolic murmur of aortic insufficiency
Focal neuro deficits

Treatment of aortic dissection

IV BB to lower BP and arterial shear stress


Noninvasive imaging procedure such as
TEE, CT angio, or MRI.

Aortic dissection is especially important to


recognize because of why

MI management such as anticoagulation and


thrombolytics can acutely worsen this
condition.
Cystic degeneration of the elastic media as in various connective tissue disorders such as Marfan and Ehlers Danlos.

What predisposes patients to aortic


dissection?

Other factors include:


HTN
Aortic valve probelms such as stenosis and congential biscuspid
Coarctation of the aorta
Pregnancy
Atherosclerotic disease
May also occur iatrogenically after cardiac surgery or cahteterization.

Complications of aortic dissection

Clinical features of aortic dissection

Intimal flap may occlude branches


HEmatoma may rupture --> cardiac tamponade or into thepleural
space causeing exsanguination
Severe aortic regurg leading to fulminant heart failure

Sudden onset of ripping or tearing pain in the chest with radiation to the back.
May also radiate to the neck or extremities as dissection extends.
Often maximal at onset (different from ischemic pain which is progressive).
Pain of dissection is also not relieved by nitrates. May also be associated with
early diastolic murmur of aortic insufficiency..

BP in aortic dissection

Most are hypertensive


If hypotensive --> rupture, cardiac tamponade, or
dissection of SCA supplying the arm.

Main neuro deficits in aortic dissection

Horner syndrome --> compression of


superior cervical ganglion
Hemiplegia --> carotid artery involvement.

Type A vs B aortic dissection

A - always involves ascending aorta, but can involve other parts 2/3 originate in the ascending aorta a few cm above aortic valve.
B - does not involve ascending aorta but can involve any other part.

Why classify Aortic Dissection into Type A


and B?

Aim of medical management of Aortic


Dissection

A - urgent surgical therapy with replacment of hte involved aorta and


sometimes the aortic valve. Without surgery mortality rate is 90%
B - Usually first managed medically, and surgery usually performed only for
complications such as rupture or ischemai of a branch artery of the aorta.

Prevent propagation of the dissection by reducing mean arterial pressure and the rate of rise of
arterial pressure
IV vasodilators such as sodium nitroprusside to lower BP to goal <120, along with IV BBs such
as metoprolol to reduce shear forces and try to achieve a heart rate of 60 BPM.
Labetalol --> both a vasodilator and a BB...

Definition of aortic aneurysm

AAA pain?

>3cm

AAA ruptue anteriorly - exsanguination within minutes


Rupture posteriorly? And if bleeding is confined to retroperitoneum,the peritoneum can produce a
local tamponade and the patient presents with severe lower back pain or midabdominal pain....
mortality rate of rupture AAA 80%, with 50% dead before they reach the hospital.

Risk of rupture based on size

What asymptomatic AAAs get fixed?

<5 cm is low
>6 cm is 10-20%

5.5cm or greater
Dacron graft in the past, but nowadays with endovascular aortic stent graft.
those below that should be screened with US or CT or MRI at 3-12 months
depending on risk of rupture.

HIV with unknown CD4 presents with subacute onset of fever, dry
cough, and gradually worsening dyspnea. No ARV or prophylactic
meds.
Tachypneic and hypoxemic

AIDS with probably Pneumocystis pneumonia


Also consider:
TB
Atypical mycobacteria
Cryptococcosis
Disseminated histoplasmosis
Also the regular old bugs:
S. pneumoniae
Mycoplasma
Viruses such as influenza

Oral thrush happens when CD4 is what?

<250.

PCP happens in HIV infected patients with


CD4 is below what

<200

Lab markers that are suggestive of PCP

Elevated LDH
Lack of sputum production

Why are ABGs helpful in PCP

Definition of AIDS

Arterial oxygen concentration <70mmHg or A-a


gradient of >35mmHg suggests a worse prognosis,
and corticosteroids may be helpful, followed by
treatment with Bactrim.

CD4<200
or
AIDS defining illness in HIV+ patient.

PCP caused by what?

Normal CD4 levels

Pneumocystis jirovecii (formerly carinii),


which is a unicellular fungus that causes
pneumonia in IC patients.
600-1500
As they decline to <500, you get immune system compromise and
patients become increasingly susceptible to unusaul infections or
malignancies.

What happens when people are first


infected with HIV?

Acute HIV syndrome --> sudden onset of mononucleosis, with


fever, H/A, LAD, pharyngitis, and sometimes a macular rash...
Then a latent period of 8-10 years

CD4<500, what pathogens

Recurrent pneumonias
TB
Vaginal candidiasis
Herpes zoster

CD4<200

PCP
Toxo
Cryptococcosis
Histoplasmosis
Cryptosporidiosis

Cd4<50

Disseminated infection like with histo or


MAC, as well as CMV: retinitis, colitis,
esophagitis, or CNS lymphoma.

What is the most common opportunistic


infection affecting AIDS patients?

PCP. Is often very difficult to diagnose. Clinical


presentation ranges from fever without respiratory
symptoms to mild, persistent dry cough, to significant
hypoxemia and respiratory compromise.

CXR findings in PCP

Definite diagnosis of PCP

Ranges from near normal CXR to diffuse bilateral


"fluffy" infiltrates to large cysts or blebs that may
rupture and cause a pneumo...
Giemsa or silver stain, which usually requires induction of sputum using aerosolized hypoertnoic
saline to induce cough or BAL to obtain diagnostic specimen.
LDH elevation may also be used but this is nonspecific and may be present in disseminated histo
or lymphoma... More useful as a negative predictor.
PCR techniques

LDH levels in PCP negative patients?

Patients with an LDH of <220 IU/L are VERY


unlikely to have PCP
Also, if CD4>250 or if they are on bactrim therapy, it
is also very unlikely.

Treatment for PCP

O2 <70mmHg or A-a gradient >35 have significant disease, and should be


given prednisone in addition to antimicrobial therapy.
Bactrim
Pentamidine
Clindamycin with primaquine

Diffuse interstitial infiltrates in an AIDS


patient

PCP
Disseminated histo
MTB
Mycobacterium kansasii
Cryptococcal lung disease too..

Cavitary lung lesions on CXR in an AIDS


patient

CXR findings in TB in AIDS patient

HIV AID patient with classically appearing


TB disease on CXR?

TB
PCP
Coccidiomyocosis
CD4>200: Bilateral apical infiltrates with cavitation.
CD4<200: CXR extremely variable.. can be hypoxemic with little
infiltrate because TB infects both alveoli and hte circulation.

Consider M. kansasii, which can cause pulm disease and radiographic findings
typicaly of MTB

MTB in AIDS patient may be more likely to spread hematogenously and


produce extrapulmonary manifestations.

Most common CNS mass lesions

Toxoplasmosis vs CNS lymphoma in AIDS


patients

Toxoplasmosis treatment

Cerebral toxoplasmosis. Typically presents with h/a,


seizures, or focal neuro deficits, and is seen on CT or
MRI scan as multiple enhancing lesiosn, often located
in the basal ganglia.
Toxo - multiple, regresses with treatment (Sulfadiazine with
pyrimethamine)
CNS lymphonam - usually solitary, does not regress

Sulfadiazine with pyrimethamine

Confirming CNS lymphoma in AIDS


patients

Usually stereotactic brain biopsy


But new researhc shows that >90% of patients with
CNS lymphoma have EBV DNA on lumbar puncture.

Cryptococcal meningitis presentation

Chronic, indolent infection which often presents


with vague sx of mood or personality changes,
H/A, or visual disturbances.

How to diagnose cryptococcal meningitis

Treatment of Cryptococcal meningitis

Serum cryptococcal antigen


LP --> frequently shows lack of inflammation, but patient does
present with elevated IC pressure. Diagnosis can be confirmed
through india ink staining, bu fungal culture, or by measuring
crypto antigen in CSF.
Induction with IV amphotericin plus Flucystosine
Then
Chronic suppression with oral fluconazole...
Also frequent LPs or shunts to treat elevated intracranial hypertension..

CMV infection presentation

Viremia with persistent fever and constitutional symptoms


Retinitis that can lead to blindness
Esophagitis that can cause severe odnynophagia
Colitis
Necrotizing adrenalitis --> which may produce clinical adrenal insufficiency

IV Ganciclovir
Foscarnet
Cidofovir

Therapy for CMV

How might Mycobacterium avium


intracellulare present?

<50 CD4
Disseminated infection with persistent fevers, weight loss, constitutional symptoms, and GI
symptoms such as abdominal pain, chronic watery diarrhea.
Diagnosed via mycobacterial blood culture

Treatment of MAC

Clarithromycin
Ethambutol
Rifabutin
For weeks.

Prophylaxis for AIDS patients

<200 --> PCP via Bactrim


<100 --> Toxo via Bactrim
<50 --> MAC via Clarithromycin 500mg BID or Azithromycin 1200mg weekly..
Prophylaxis can be d/c if HAART is started and the patient's CD4 levels recover.

HAART, what does it consist of

Two Nucleoside Reverse Transcriptase Inhibitors


Along with:
either
NonNucleosideReverse Transcriptase Inhibitor
or
Protease inhibitor

Initiation of HAART for whom

Acute HIV infection


CD4<500
Pregnancy
Symptoamtic patient regardless of CD4

Why not start HAART in acutely ill patient

Side effects
Also, within 1-2 weeks of starting HAART, improvement in the
immune system can actually cause worsening symptoms as a result
of host responses --> Immune Reconstituion Inflammatory
Syndrome or IRIS.

Treatment of Toxoplasmosis

PCP Presentation

Sulfadiazine with
Pyrimethamine

Fever, dry cough, sometimes hypoxemia.


CD4<200 usually
CXR may be normal but may have faint bilateral infiltrate
Elevated serum lactic acid dehydrogenase level.

MAC prophyalxis

When CD4<50
Clarithromycin or
Azithromycin

Warning sign of possibly critical limb


vascular insufficiency

Rest pain
pallor
Pulselessness

Peripheral Arterial Disease prevalence

16% of americans
May exist w/o clinically recognized coronary or cerebrovascular
disease..
PAD also confers same risk of death as in persons with prior MI or
CVA.

Most important risk factors for PAD

Smoking
DM
HTN, HLD, elevated homocysteine also play significant roles.

Most common site of claudication pain

Calves
Proximal stenosis may produce pain in buttocks or thighs..
Severe occlusion may produce rest pain, which often occurs at night and may be relieved by sitting
up and dangling the legs, using gravity to assist blood flow to the feet.

Physical exam maneuver that may help


diagnose PVD

PE findings in the skin suggestive of PVD

PVD ABI values

Elevation of feet above heart level may produce


pallor of the soles. If placed in a dependent
position --> rubor as a result of reactive hyperemia.

Hair loss on the legs and the feet --> thickened,


brittle toenaisl, shiny atrophic skin.
Severe ischemia --> ulcers or gangrene.

Claudication typically 0.41 to 0.9


Rest pain <=0.4

PAD mortality rate

50% over next ten years, mainly due to


likelihood of coexisting atherosclerotic
disease.

Single most important risk factor for PVD?

Smoking. Cessation reduces risk of fatal or nonfatal MI by as much


as 50%, more than any other mdical or surgical intervention

Drugs for PVD?

Pentoxifylline (xanthine derivative that increases RBC elasticity) may help, but
it may not.
Cilostazol - Phosphodiesterase inhibitor with vasodilatory and antiplatelet
properties. Has been shown to improve maximum walking distance.

Definition of critical leg ischemia for PVD

ABI<0.4
Severe or disabling claudication
Rest pain
Nonhealing ulcers
These people should be evaluated for revascularization procedures.

Less common cause than atherosclerosis


for chronic peripheral arterial
insufficiency?
Thromboangiitis obliterans aka, and what
is it

Fibromuscular dysplasia who gets this

Takayasu arteritis

Person with longstanding PVD who


presents with sudden unremitting pain

Signs of acute arterial occlusion

Thromboangiitis obliterans or Buerger


disease

Buerger disease
Inflammatory condition of small and medium-size arteries that may
affect upper and lower extremities and is found almost exclusively
in smokers, especially males younger than 40 yo.

Hyperplastic disorder affecting medium and small


arteries, usually occurs in women. Generally renal or
carotids are involved... but can affect the limbs

Affects branches of the aorta, most commonly the


SCAs, and causes arm claudication and Raynaud
phenomenon, along with constitutional sx such as
fever and weight loss.
Acute arterial occlusion, most commonly the result of embolism
from the heart, or in situ thrombosis...
Conditions that may cause embolization: AF, DCM, Endocarditis

Six Ps:
Pain
Pallor
Pulselessness
Paresthesias
Poikilothermia
Paralysis
First five develop quickly
Paralysis will develop if arterial occlusion is severe and persistent.

Acute arterial occlusion management

Takayasu arteritis

Anticoagulation with heparin to prevent propagation of thrombus


Affected limbshould be placed below horizontal plane
Angiography
Surgical removal may be performed.
or Intraarterial TPA directly into the thrombus...

Really an aortitis
Fever
Stenotic lesions of the SCAs, which may cause unequal blood
pressures, diminished pulses, and ischemic pain in the affected
limbs.

Chronic incomplete arterial occlusion may


cause what

Exertional pain or fatigue


Pallor on elevation of the extremity
Rubor on dependency.

Signs of hypertensive retinopathy

Narrowing of arteries
Arteriovenous "nicking"
Flame shaped hemorrhages with cotton wool
exudates

Labs to get for the hypertensive patient

Renal function: electrolytes creatinine and U/A


SErum glucose
Lipid profile
Baseline ECG

Prehypertension

120-139
80-89

HTN Stage I

140-150
90-99

HTN Stage II

>160
>100

Prevalence of secondary HTN along all


cases of hypertension

5-20%

When should a secondary cause of HTN be


considered?

<25 yo or >55
Malignant HTN presentation
Refracotry HTN requiring >=3 antihypertensive meds
HTN that has suddenly become uncontrolled
Rising Cr level with use of ACEIs, or other clinical signs of a secondary cause.

Primary aldosteronism
Cushing syndrome
Pheochromocytoma
Hyperthyroidism
Acromegaly (excess GH)

Endocrine causes of HTN

Renal disease causes of HTN?

Misc causes of HTN

Can be parenchymal --> glomerulonephritis, PCKD,


renal tumors
Can also be renovascular --> atherosclerosis, or
fibromuscular dysplasia
OSA
Coarctation of aorta
Increased intravascular volume (posttransfusion e.g.)
Hypercalcemia
Medciations (NSAIDs, sympathomimetics, glucocorticoids, high dose estrogens).

Target organ damage of hypertension

Cardiomyopathy
Nephropathy
Retinopathy
Cerebrovascular disease

Prehypertension interventiosn

Lifestyle modifications

Stage I HTN

Single antihypertensive

Stage II HTN

Two antiHTNs in combination

Young vs old antihypertensive choice

Generally,
young - BBs and ACEIs
Old - CCBs, thiazides
AAs? Thiazides better than BBs, and ACEIs and ARBs may be less effective
than in white patients.

Target blood pressure for anti-HTN


therapy

135/85, unless patient has DM or renal


disease, in which case target is 130/80.

JNC recommendations for choice of


antihypertensive

Low dose thiazides are a good first choice


in uncomplicated HTN

Clonidine side effects

Postural hypotension
Drowsiness
Dry mouth
Rebound HTN with abrupt withdrawal

BB side effects

Bronchospasm
Hyperlipidemia
Depression
Erectile dysfunction

Potassium sparing diuretic side effects

Hyperkalemia
Gynecomastia

Hydralazine side effects

Reflex tachycardia
H/A
Angina
Lupus-like syndrome

ACEI side effects

Hyperkalemia
Cough
Angioedema
Acute renal failure
Orthostatic hypotension

CCB side effects, dihydropyridines

Amlodipine or Nifedipine
Tachycardia, flushing, GI side efects, hyperkalemia
edema

CCB side effects, nondihydropyridines

Diltiazem
Verapamil
Heart block, constipation

What antihypertensives are CI in


pregnancy

ACEIs
ARBs

Agent of choice in hypertensive patient


with heart failure

ACEI

Agent of choice in hypertensive patient


with DM

ACEI

Man with hypertension and BPH

Alpha blockers such as Carvedilol

Hypertension and CAD, first line choice

Beta blockers

Most common cause of secondary


hypertension?

Helpful in diagnosing renovascular disease

Renal causes: parenchymal or renovascular...


Potassium level may be low or borderline low in
renovascular disease 2/2 seconadry hyperaldosteronism.
Captopril enhanced radionuclide renal scan
MRA
Spiral CT
Surgical or angioplastic correction may be considered.

Clinical findings of polycystic kidney


disease

Drugs that may cause hypertension

OSA eventually leads to what

Hyperthyroidism presentation

AD --> FHx
Bilateral flank masses
Flank pain
Elevated BP
Hematuria
Anabolic steroids
Sympathomimetic drugs
TCAs
OCPs
NSAIDs
Illicit drugs --> cocaine
Caffeine
EtOH

Hypoxic and hypercarbia --> systemic


vasoconstriction, systolic hypertension,
and pulmonary hypertension.
Widened pulse pressure with increased SBP and decreased DBP
Hyperdynamic precirdium
Warm skin
Tremor
Thyroid gland enlargement or a nodule
Low level of TSH, high T4 are diagnostic.

Glucocorticoid excess states

Thinning of extremities with truncal obesity


Round moon facies
Supraclavicular fat pad
Purple striae
Acne
Psychiatric symptoms
Excess of glucocorticoids can cause secondary HTN because these may also have mineralocorticoid activity.

Carcinoid syndrome presentation

Cutaneous flushing
Headache
Diarrhea
Bronchial constriction with wheezing
Often, HTN

Diagnosis of HTN requires what

Hypertensive encephalopathy, diagnosis of


what?

How might Hypertensive encephalopathy


present?

>2 or more blood pressure measurements across at least two visits

Exclusion. Altered mental status + severely elevated blood


pressures...
Should rule out stroke, bleed, meningitis, or mass lesions.... or
drugs such as cocaine and amphetamines...

Severely elevated BP
confusion
Increased ICP
And/or seizures
Must rule out whole host of other things including drugs, stroke, SAH, meningitis, mass lesions,
etc.

Treatment for Hypertensive


Encephalopathy

PArenteral medications should be used to loewr BP


to 160/100-110 range. NOt to normal!!!

Men2A vs Men2B

2A - pheo, medullary thyroid cancer, hyperparathyroidism


2B - pheo, medullary thyroid cancer - mucosal neuromas.

Examples of acute end organ damage

Hypertensive encephalopathy
Myocardial ischemia or infarction assoc with markedly elevated BP
Aortic dissection
Stroke
Declining renal funtion with proteinuria
Pulm Edema --> acute left vetnricular failure
Retinopathy

Hypertensive urgency treatment

Loweroing over 1-2 days, can be monitored in


outpatient setting if f/u is reliable.

Pathophys of hypertensive encephalopathy

Pressure overcomes cerebral vasoconstriction --> as MAP increases beyond


normal range, endothelial dysfunction, with increased permeability of the BBB,
leading to vasogenic edema and formation of microhemorrhages...
Then you get lethargy, confusion, headaches, vision changes.

Imaging findings of Hypertensive


Encephalopathy?

MRI --> Posterior leukoencephalopathy, usually in


the parietooccipital regions
CT scanning may or may not come up with anything...

What happens if Hypertensive


Encephalopathy is not treated?

badness: seizures, coma, and death.

Can you set hypertensive emergency at


given BP?

Treatment of hypertensive emergencies

No. Autoregulation failure can occur in normotensive patients at BPs as low as 160/100.
Longstanding HTN --> adaptive mechanisms such as cerebral arterial autoregulation, may not
show clinical manifestations until BP>220/110.
Thus, should focus on symptoms such as hypertensive encephalopathy rather than the numbers.

IV sodium nitroprusside (instant, and titratable)


Monitoring with arterial catheter is often necessary
Problem with nitroprusside is that metabolite may accumulate when given for 2-3 days --> cyanide or thiocyanate toxicity
when vasodilators such as nitroglycerin decrease preload..
May be used with BB to blunt reflex tachycardia.
IV Labetalol for dissection.
Acute pulm edema --> nitroglycerin
MI or infarction --> IV nitroglycerin

Acute cerebral infarction in the setting of


very high BP?

Do not lower BP, because of the possibility


of worsening cerebral ischemia
(permissive BP).

Other nonclassical symptoms of pheo

Episodic anxiety
Tremor
Orthostatic hypotension caused by volume
contraction from pressure-induced natriuresis

How many cases of HTN result from pheo?

0.01-0.1%

Diagnosis of pheo

24 Urine for metanephrines,


vanillylmandelic acid, and catecholamines

If gross scanning does not pick up a pheo...

Scintigraphic localization with 123Imetaiodobenzylguanidine or Octreotide


scan.

What should be done before a pheo


surgery is performed?

Phenoxybenzamine started a week prior.


Liberal salt diet to expand contracted blood volume...
Beta blocking agent but only after alpha blockade is established.

Pheo gene mutations

RET in MEN2
VHL gene in Von-Hippel Lindau
Familial pheo
Neurofibromatosis

Beta blockers in pheochromocytoma

Can paradoxically increase BP b/c of


unopposed alpha adrenergic effects

Transaminase levels >1000

Viral hepatitis progression

Extensive hepatic necrosis such as toxic injury, viral hepatitis, and ischemia (shock liver).
Alcoholic hepatitis has levels <500 almost always, and AST/ALT ratio of 2:1

Prodrome of nonspecific constitutional sx including fever, nausea, fatigue,


arthralgias, myalgias, headache, and sometimes pharyngitis and coryza.
Followed by onset of visible jaundice caused by hyperbilirubinemia, with
tenderness and enlargement of the liver, and dark urine caused by
bilirubinuria.

Who is most susceptible to HEV infection

Pregnant women --> severe hepatic necrosis and fatal liver


damage.

HBV usually transmitted how?

Sexually
IVDU - blood
Birth chronically infected mothers...

Outcome of HBV infection depends mostly


on what?

Age of infection:
Birth --> 90% develop chronic infection
Adults - 95% recover completely without sequelae, 5-10% develop
chronic hepatitis, with fewer developing cirrhosis...

Acute liver failure, top cause?

Toxin or drug induced liver injury... acetaminophen, or Amanita


phalloides, or due to idiosyncratic reactions to drugs like
halothane, isoniazid, phenytoin.
Direct toxin are dose related, but idiosyncratic ones are not.

Acute hepatic failure characterized by what

Rapid progression of encephalopathy from confusion or


somnolence to coma. Patients may also have worsening
coagulopathy as measured by increasing PT times, rising bilirubin
levels, ascites and peripheral edema...

Mortality rate for comatose fulminant


hepatitis

80%. Is often fatal wtihout an emergency


liver transplant.

What is found during window period

Anti-HBc IgM
Anti-HBc IgG
Anti-HBe

Acute HCV infection

Rarely diagnosed
Not HCV antibody
But rather HCV RNA assay

Chronic hepatitis B treatment

Interferon
Lamivudine (nRTI) at a level lower than when it is
used for HIV.

Chronic HCV treatment

Pegylated interferon
Ribavirin
Protease inhibitor if genotype I.

How much acetaminophen for toxicity?

10g or more usually, but lower doses may cause injury in patients with preexisting liver disease,
particularly in those who abuse alcohol.
Alcohol augments P450 activity which produces more toxin from acetaminophen, also
phenobarb does the same...
Less glutathione is available in alcoholism, malnutrition, or AIDS...

Treatment for acetaminophen toxicity

Decontamination with charcoal


Administration of N-acetylcysteine (provides cysteine to replenish glutathione stores).
N-acetylcysteine should be stared withint first 10 hours, and continued for 72...
NO meds that have any sort of hepatotoxic functioning.

Antiretroviral prophylaxis for needle sticks


involving HIV

Lamivudine and others.

Presence of what indicates prior infection


with HBV

Anti-HBc IgG

What is usually positive during window


period of acute infection with HBV?

Anti-HBc IgM

Primary vs secondary amenorrhea

Primary - no menses in a girl by age 16


Secondary - normally menstruating woman stops
having periods for 3 consecutive months

Largest single cause of oligomenorrhea

Polycystic ovarian syndrome - 30% of all cases.

PCOS history / lab / therapy

Irregular menses since menarch, obesity, hirsutism


Slightly elevated testosterone, elevated LH/FSH
OCPs

Common causes of oligomenorrhea

PCOS
Hypothyroidism
HyperPRL
Ovarian failure
Sheehan syndrome

How are thyroid function and galactorrhea


connected?

In primary hypothyroidism, TRH in


increased. TRH also stimulates prolactin
secretion...

What are the most common functional


pituitary tumors in both men and women?

Prolactinomas
If PRL>200 ug/L, an MRI is indicated.

Most common cause of hypothyroidism in


the US?

Lymphocytic (Hashimoto) thyroiditis, in which cytotoxic antibodies are


produced, which leads to thyroid atrophy and fibrosis.

Hyperprolactinemia inhibits GnRH secretion, leading to


amenorrhea in females, infertility and diminished libido in men.

Next?
Surgical or radioactive iodine treatment for hyperthyroidism

Worldwide cause of hypothyroidism

Iodine deficiency --> Goitrous


hypothyroidism. In the US this is rare.

Common sx of hypothyroidism

Fatigue
Weight gain
Muscle cramping
Cold intolerance
Hair thinning
Menstrual changes
Carpal tunnel syndrome

Emergency hypothyroidism?

When is there excess Thyroid binding


globulin?

After generalized myxedema. May also have enlarged heart,


nonmechanical intestinal obstruction, and delayed relaxation
phase of DTRs. Without treatment may become stuporous and
hypothermic --> emergency!!! Treat aggressively with IV
levothyroxine.

Pregnancy and oral contraceptives...


Nephrotic syndrome --> may be low...

Subclinical hypothyroidism?

TSH mildly elevated (4-10) but free T4 is within normal range. Patients may be asymptomatic or report
vague and subtle symptoms of hypothyroidism such as fatigue/. About half will progress to overt
hypothyroidism within 5 years.
They often also have some sort of derangement of cholesterol metabolism. Elevated total and LDL...
Thyroid replacement can be inititated.... esp if postiive antithyroid antibodies are present.

Dosing of levothyroxine

100-150 ug per day


Older patients? STart lower at 25-50 ug/day, increase at increments over 4-6 weeks until patient reaches a euthyroid state...
Overly rapid replacement can overwhelm coronary or cardiac reserve. Goal of treatment is normalized TSH, ideally in lower
half of the reference range...
TSH level wil take 6-8 weeks to readjust to new dosing level, so f/u lab testing should be scehduled accordingly... Patients
may not get full relief until 3-6 months after normal TSH is achieved.

TSH elevation with rubbery smooth


enlarged thyroid

C. Hashimoto thyroiditis

What might be positive during the window


period?

Anti-HBc IgG

Hemochromatosis, what labs

High ferritin
High transferrin saturation

HCV who develops chronic infection,


cirrhosis, HCC?

70-80% of infected develop chronic


20% of those with chronic develop cirrhosis after 20
years.
1-4% of cirrhotics will develop HCC

Side effects of HCV infection treatment

Interferon --> influenza-like syndrome and


depression
Hemolysis with ribavirin

Treatment for portal HTN

Nonselective BBs (propranolol) lower portal


pressure
IV octreotide causes splanchnic vasoconstriction.

Treatment for ascites

Sodium restriction
Sprinolactone
Loop diuretics
Large volume paracentesis

Spontaneous bacterial peritonitis


treatment, how to diagnose

Presentation of spontaneous bacterial


peritonitis

Treatment for spontaneous bacterial


peritonitis

Ascitic fluid contains >250 neutrophils/mm^3


And/or
Confirmed with positive culture, most commonly:
E. Coli
Klebsiella
other enteric flora
Enterococci

Abdominal pain, distention, fever, decreased bowel


sounds, or sometimes few abdominal symptoms, but
worsening encephalopathy

IV antibiotics such as:


Cefotaxime
Ampicillin/Sulbactam (Unasyn)

Why do people with cirrhosis get spider


angiomata, palmar erythema, testicular
atrophy, and gynecomastia?

Decreased liver production of steroid hormone binding globulin --> increase in unbound estrogen.

How might variceal bleeding present?

Massive hemorrhage
OR
More subtle bleeding that can trigger bout of
encephalopathy.

Treatment IV for bleednig varices?

Octreotide to cause splanchnic vasoconstriction and


reduce portal pressure.

What characterizes hepatic


encephalopathy

Mental status changes


Asterixis
Elevated ammonia levels

Treatment for hepatic encephalopathy

Lactulose
Antibiotics such as neomycin (kill bacteria
that make ammonia??)

Low gradient ascites

Peritoneal carcinomatosis
Tuberculous peritonitis
Pancreatic ascites
Bowel obstruction or infarction
Serositis --> lupus
Nephrotic syndrome.

IV therapy for spontaneous bacterial


peritonitis

IV ampicillin, gentamicin
Third generation cephalosporin

Hepatic disease affecting other organs?

Hepatorenal syndrome
Hepatopulmonary syndrome

Hepatorenal syndrome, what happens

Typically presents as progressive decline in renal function in


patients with significant ascites. Pathogenesis is poorly
understood, but appears to involve renal vasoconstriction...
Treatment difficult,prognosis poor. Liver transplant.

MELD score is based on what

Serum bilirubin
Serum creatinine
INR
Repalces the Child-Pugh system... A to C.

What is sclerosing cholangitis

PSC
Autoimmune destruction of both intrahepatic and extrahepatic bile ducts and is often associated
with inflammatory bowel disease, most commonly ulcerative colitis.
Patients present with jaundice or symptoms of biliary obstruction. Cholangiography reveals
characteristic beading of the bile ducts.

IBD is assoc with what liver disease

Primary Sclerosing Cholangitis


Beading of the bile ducts.

Ranson criteria for the severity of


pancreatitis

Most common causes of acute pancreatitis

Initial
Age>55
WBC>16k
Serum glucose>200
Serum LDH>350
AST>250
Within 48h of admission
Hct drop>10
BUN rise >5mg/dl after IV hydration
Arterial PO2<60mmHg
Serum calcium <8mg/dl
Base deficit >4mEq/L
Estimated fluid sequestration of >6L

Gallstones most common (30-60%)


When >=3 Ranson criteria are present, a more complicated course can be predicted, usually with pancreatic necrosis.
Alcohol (15-30%)
Hypertriglyceridemia (1-4%) and occurs when serum triglyceride
levels>1000 as seen in familial dyslipidemias or diabetes...

Iatrogenic acute pancreatitis

ERCP. 5-10% of such procedures.... Any


procedure that manipulats the gland... or
even blunt abdominal trauma.

What happens when patient has "idiopathic"


pancreatitis that does not occur in the
context of alcohol or gallstones?

Number one cause is still biliary tract


disease, either biliary sludge (microlithiasis),
or sphincter of Oddi dysfunction.

Drugs that may trigger acute pancreatitis

Antiretroviral didanosine
Pentamidine
Thiazides
Furosemide
Sulfonamides
Azathioprine
L-asparaginase

Infections assoc with triggering acute


pancreatitis

Mumps
CMV

Pain of acute pancreatitis, exacerbating or


relieving factors?

Relieved by sitting up and bending forward


Exacerbated by food (also have N/V).

Why are people with acute pancreatitis


volume depleted?

Vomiting
Inability to tolerate PO intake
Third spacing due to inflammatory processes, with sequestration of
large volumes of fluid within the peritoneal cavity.

Signs of hemorrhagic pancreatitis

Periumbilical ecchymoses (Cullen sign)


Flank ecchymosis (Grey Turner sign)
This is due to blood tracking along fascial planes.

Problems with serum amylase

How long does amylase stay elevated?

Treatment of acute pancreatitis

Nonspecific, can be elevated in other abdominal processes such as


GI ischemia with infarction or perforation. Even just the vomiting
assoc with pancreatitis can cause elevated amylase of salivary
origin.

3-4 days after initial pancreatitic attack

Mainly supportive
--> "Pancreatic rest"
With-holding food or liquids by mouth until symptoms subside, and adequate narcotic analgesia, usually with meperidine
which is a fast acting opioid.
IV FLUIDS!!!

When can someone with pancreatitis get


restarted on PO

When pain has subsided and patient has


bowel sounds, stat on oral clears and
advance as tolerated.

Most common cause of early death in


patients with acute pancreatitis

Hypovolemic shock --> multifactorial, third spacing with sequestration of large fluid volumes in
the abdomen as well as increased capillary permeability.

Complications of acute pancreatitis

Phlegmon --> solid mass of inflamed pancreas which may become necrotic or
infected... abscesses --> 100% mortality if not drained.

Others develop ARDS or myocardial dysfunction which both result in a pulmonary edema.

Pseudocyst --> can cause pain, large or expanding or become infected, they
usually require drainage. Usually resolve in 6 weeks if smaller than 6 cm.

When do pancraetic abscesses following


acute pancreatitis develop?

2-3 weeks after onset of pancreatitis

Presence of gallstones vs incidence of


symptoms

10% of people with gallstones will develop


symptoms within ten years.

Biliary colic

Sudden onset, often precipitated by a large or fatty meal, with severe steady pain in the RUQ or
epigastrium, lasting b/w 1-4 hours....
Mild elevatiosn of alkaline phosphatase level, and slight hyperbilirubinemia... but elevations
>3g/dl suggest a common duct stone.

Signs on US of acute cholecystitis

Gallbladder wall thickening


Pericholecystic fluid
Clinical: persistent RUQ abdominal pain, with fever and leukocystosis. Cultures
of bile in the gallbladder --> enteric flora such as E. coli and Klebsiella.

HIDA scan

Hepatobiliary iminodiacetic acid scan.


Positive test shows visualization of liver by
the isotope.

Treatment of acute cholecystitis

Nil per os
IV fluids and antibiotics
Early cholecystectomy w/i 48-72 hours

Cholangitis

Intermittent obstruction of bile duct --> reflux of bacteria up biliary tree, followed by purulent
infection behind obstruction.
Sepsis? Urgent decompression of the biliary tree, either surgically or by ERCP, to remove stones
endoscopically after performing a papillotomy which allows stones to pass.

Initial ranson signs of badness

Age>55
WBC>16k
Serum glucose>200
Serum LDH>350
AST>250
Note that this does NOT include amylase or lipase.

Ranson signs of badness w/i 48 hours of


admission

Fever, RUQ pain, and hx of gallstones

Hct drop >10 points


BUN rise in 5 after IV hydration
PO2<60
Serum calcium<8
Base deficit >4
Estimated fluid sequestration of >6 L

Acute cholecystitis.
NOT acute cholangitis

Pancreatic pseudocysts presentation

Abdominal pain, mass, persistent


hyperamylasemia in a patient with prior
pancreatitis.

Three most common causes of acute


pancreatitis in the US

Gallstones
Alcohol consumption
Hypertriglyceridemia

Ranson criteria how many signifies


significant mortality

>=3

Third degree AV block resulting in syncope


with widened QRS, treatment?

Calciphylaxis.. what is this

Indicates that defect / block is below AV node, so atropine or


isoproterenol cannot be used typically...
Temporary transcutaneous or transvenous pacemaker and eval for
placement of permanent pacemaker.
Characterized by systemic medial calcification of the arteries, i.e.
calcification of the tunica media... Most common in Stage V hemodialysis
patients, with ESRD and 2/2 severe hyperparathyroidism...
Rare disorder... Can mimic cellulitis.

Risk factors for calciphylaxis

Shy-Drager syndrome

Causes of orthostatic hypotension

ESRD pateints on warfarin, Calcium


binders, etc...

aka Multipel system atrophy, may result in


orthostatic hypotension and movement disorders
in addition to bladdder problems.
Drug induced
Peripheral neuropathy
Idiopathic postural hypotension
Neurologic disorder: Shy Drager syndroem
Physical deconditioning
Sympathectomy
Acute dysautonomia (GBS variant)
Decreased blood volume (adrenal insufficiency, acute blood loss, etc.)
Carotid sinus hypersensitivity

Most common cause of syncope

Vasovagal syncope --> excessive vagal tone causing impaired


autonomic responses. Episodes often ar eprecipitated by physical or
emotional stress or by a painful experience.

Actions that increase vagal tone and may


lead to syncope

Micturition
Defecation
Coughing

Vasovagal syncope needs to be


differentiated from what

Orthostatic hypotension

Who gets carotid sinus hypersensitivity?

Older men, episodes can be triggered by turnign head to side, wearing a tight collar, or even by
shaving the neck over the area. Pressure over carotid sinus --> excess vagal activity which leads to
bradycardia, fall in CO, can produce --> sinus arrest or even an AV block... Sometimes, but less
commonly, this can cause a fall in arterial pressure without a drop in HR.
Recurrent syncope? Pacemaker.

Orthostatic hypotension, causes?

Hypovolemia
Imapired autonomic responses

Neuro causes of orthostatic hypotension

Diabetes
Chronic idiopathic orthostatic hypotension
Primary neurologic conditions

What suggest factitious symptoms related


to syncope

Multiple events that are all unwitnessed,


or that occur only in periods of emotional
upset.

Syncope caused by cardiac outflow


obstruction typically presents when?

During or immediately after exertion.


These include aortic stenosis and HCOM

Most common cardiac cause of syncope?

Arrhythmias, usually of the brady type, most often


due to degenerative SA node dysfunction and AV node
blocks.

Sick sinus syndrome

Elderly patients, one of the most common indications for pacemaker


placement. Patients wtih SSS may experience sinus bradycardia or arrest,
alternative with SVT, most often AFib (tachy-brady syndrome).
Tachyarrhythmias such as Afib or flutter, SVT, VT, or FV are more likely to
produce palpitations than syncope.

Three main types of heart block

First degree --> prolonged PR interval (>200ms), which also has a conduction delay. Prognosis good, no need for pacing.
Second degree -->
--> I. Progresive lengthening of PR interval until skipped beat. No need for pacing unless symptomatic.
--> II. Dropped beats without lengthening. Permanent pacing indicated because usually progresses to type III.
Third degree --> complete heart block. SA node and AV node fire at independent rates. Atrial rhythm is faster than
ventricular escape rhythm. Permanent pacing is indicated

Mobitz II is typically the result of what

Block within the bundle of His. Later may progress to


complete heart block, thus permanent pacing is
required.

Mobitz I --> bradycardia with symptoms,


first drug choice?

Atropine

Presentation of ulcerative colitis

Abdominal pain and frequent, sudden urge to defecate resulting in small voluem with blood or
mucus stooling

Symptoms of colitis

Crampy abdominal pain with tenesmus, low volume bloody mucoid


stool, colonic dilatation on XR.

What must you exclude when making a


diagnosis of ulcerative colitis?

Infection: E. histolytica, Salmonella, Shigella, E. coli,


Campylobacter
C. diff as well --> C. diff can occur in the absence of
antibiotic exposure.

Life threatening complications of colitis?

Perforation or toxic megacolon

XR findings in colitis

Colonic dilatation on XR

DDx for colitis

Ischemic
Infectious (C diff, E coli, Salmonella, Shigella, Campylobacter) -->
Travel or antibiotics
Radiation colitis
IBD (Crohn disease vs ulcerative colitis)

IBD age of onset

15-25
also second peak for Crohn disease b/w ages of 6070

IBD presentation

Low grade fever


Anemia either due to iron deficiency from chronic GI blood loss or anemia of chronic disease.
May also report fatigue and weight loss.

Sx diff between UC and CD

UC: grossly bloody stool


CD: much more variable, mainly chronic
abdominal pain, diarrhea, weight loss.

CD findings in the anus

Anal fissures
Nonhealing ulcers

Radiographic findings in CD vs UC

CD: string sign on barium XR


UC: Lead pipe colon on barium XR

Colon cancer increase in CD vs UC

CD: Slight increase


UC: marked increase

Skin manifestations of CD vs UC

Both have erythema nodosum


Pyoderma gangrenosum is rare in CD but
1-12% in UC

Ocular complications of CD vs UC

Both hae uveitis commonly

Hepatobiliary complications of CD vs UC

CD:
Cholelithiasis fatty liver: common
PSC: rare
UC:
Fatty liver is common
PSC rare, but less rare than in Crohn's.

Definition of toxic megacolon

Colon dialtes to diameter >6cm. Usually accompanied by fever, leukocytosis,


tachycardia, and evidence of severe toxicity, such as hypotension, or altered
mental status.
Most serious complication: colonic perforation complicated by peritonitis or
hemorrhage.

Therapy for toxic megacolon

Centered around reducing chce of perforation and the complications thereof:


IV fluids, NG tube suction, NPo
IV antibiotics
IV steroids
Usually ineffective and usually proceeds to surgery.

colonoscopy recommendations for UC

Annual or biennial colonoscopy beginning 8 years


after diagnosis, and random biopsies should be sent...
If colon cancer or dysplasia is found --> colectomy.

Presentation of PSC

Assoc with IBD in 75% of cases


RUQ pain, jaundice, pruritus. No fever, and normal WBC count.
ERCP --> multifocal strictures of both intrahepatic and extrahepatic
bile ducts with intervening segments of normal and dilated ducts.

Evidence of diabetic retinopathy

Dot hemorrhags and hard exudates

Next step after AKI

Urinalysis and urine electrolytes to


determine whether the process is prerenal,
renal, or post renal (less likely).

Relationship between Cr and GFR

Cr always lags behind GFR

Definition of anuria vs oliguria

Anuria <50ml UO in last 24h


Oliguria <400ml UOP. Physiologically, it is the lowest amount of urine a person a normal diet
can make if he or she is severely dehydrated and does not retain uremic waste products.
Oliguria is worse than anuria... they have higher mortality rates and less renal recovery than do
patients who are nonoliguric or anuric.

Elevated BUN w/o symptoms?

Azotemia

Nephrotic syndrome may cause what

Prerenal KI

Meds that may trigger prerenal KI

ASA
NSAIDs
ACEIs

Postrenal failure by crystals?

In the intratubular region, may cause a


postrenal failure just the same as ureters
or whatever.

Most common cause of obstructive


nephropathy

Ureteral obstruction due to malignancy


Prostatic obstruction due to benign or malignant
hypertrophy...

Most common causes of acute tubular


necrosis

Nephrotoxic agfents such as AGs, radiocontrast, chemotherapy

Most common causes of


glomerulonephritis

Postinfectious
Vasculitis
Immune complex diseases (lupus, MPGN, cryoglobulinemia)
Cholesterol emboli syndrome
HUS/TTP

Medications that may cause


tubulointerstitial nephritis

Cephalosporins
Methicillin
Rifampin

Ischemic events due to hypotension, vascular catastrophe.

Also infections such as pyelo or HIV can do this.

Signs of tubulointerstitial nephritis on U/A


and microscopy

Urine that isothenuric


mild proteinuria
Leukocytes, white cell casts, and urinary eosinophils.

FENa in glomerulonephritis

Indications for dialysis in AKI

<1%

Fluid overlaod --> pulm edema


Metabolic acidosis
Hyperkalemia
Uremic pericarditis
Severe hyperphosphatemia
Uremic symptoms

Removal mechanisms for high K

Dialysis
Furosemide
Kayexalate (but not really)

Urinary sodium total level consistent with


prerenal injury

U sodium<20mEq/L

High uremia, effects on the heart?

Uremic pericarditis --> elevated jugular venous pressure, may


progress to tamponade.

Anuric patient, what must be decided?

If patient has obstructed kidneys or if


vascular supply is interrupted.

Indications for urgent hemodialysis

Uremic pericarditis
Hyperkalemia
Metabolic acidosis
Severe hyperphosphatemia
Volume overload refractory to medical management

Nonexertional chest pain that is relieved


by sitting forward

Acute pericarditis caused by SLE

Pericardial friction rub, what does this


sound like

Harsh, high pitched, scratchy sound with variable


intensity, usually best heard at the left sternal
border by auscultation.

Chest pain of acute pericarditis

Sudden onset of substernal chest pain, which worsens on inspiration and with recumbency, that
often radiates to the trapezius ridge.
Is often improved by leaning forward and exacerbated by recumbency.

Most patients get pericarditis witth what?

Friction rub, sens and spec for pericarditis

Viral infection, and they often present with


low grade fever, malaise, or upper
respiratory illness.
HIGH SPECIFICITY!
Low sensitivity, because these rubs tend to come and go over hours.
Best heard at Left Sternal Border, can have components of all three
(presystolic, systolic, and diastolic). If monophasic, can be hard to distinguish
from a harsh murmur.

Acute pericarditis EKG changes

ST segment elevation that is diffuse, in limb leads as well as precordial


May also have PR segment depression (not seen in acute MI usually) and no
reciprocal ST segment depression changes (except sometimes this is not the
case). QRS changes are also usually absent.
Also is not associated with arrhythmias!!

Prognosis and treatment of acute


pericarditis

Complciatiosn of pericarditis

Can pericarditis have both an effusion and


a pericardial friction rub at the same time?

Diagnostic criteria for SLE

Major complication of SLE

Most with acute viral or idiopathic --> treatment is symptomatic with


ASA or NSAIDs such as indomethacin.
Colchicine or corticosteriods may be used fo refractory symptoms, but in
most patients, symptoms typically resolve within days to 2-3 weeks.

Effusion and bleeding, with the worst


complication being tamponade.

Yes. It is a misconception that they do not. Important to monitor patients for


signs of developing hemodynamic compromise.

4 of 11 criteria:
Malar rash
Discoid rash
Photosensitivity
Oral or vaginal ulcers
Arthritis
Serositis
Renal involvement
Neurologic disorder: seizure or psychosis
Hematologic disorder: anemia or leukopenia, anemia is hemolytic
Positive dsDNA, Smith Ab, Antiphospholipid antibody, antinuclear, ANA must be positive in absence of drugs knownto cause
+ANA

Renal involvement, and this is what caused most of the deaths. But now it is treated with high doses of
powerful immunosuppressants
CNS disorders are also very serious, as are infection (iatrogenic due to immunosuppression), and vascular
disease --> MI.

Arhtirtis and pericarditis? usually self limited.

Sclerodactyly seen in SLE?

Pulsus paradoxus

No... seen in scleroderma.

Normal drop in BP during inspiration is <10mmHg.


When drop is larger, then pulsus paradoxus. Sign of
cardiac tamponade or other stuff.

SLE may cause false positive what ID test

Acute renal failure, hypertension, edema,


and hematuria in a young man

VDRL test

Glomerular injury... either PSGN, or IgA nephropathy.


Confirm broad diagnosis of acute GN through examiniation of freshly spun
urine sample for active sediment (Cell components, red cell casts, dysmorphic
red cells).... If present, these are signs of inflammation and establish the
diagnosis of acute GN

When are complement levels low in the


setting of GN?

SLE
MPGN
Infective endocarditis
PSGN/PIGN
Cryoglobulin-induced GN

Wegener positive with what

cANCA

pANCA is positive in what

Microscopic polyangiitis
Churg-Strauss

When might you seen antiGBM


antibodies?

AntiGBM GN
Goodpasture's

What is associated with cryo-induced GN

HCV and HBV. You might get these titers, or you


might also get cryoglobulin titers.

How can hematuria be classified?

Intrarenal - dysmorphic fragmented RBCs or


red cell casts --> glomerular origin
Extrarenal

Lab tests when working up a hematuria

Causes of intrarenal hematuria

Dysmorphic/fragmented RBCs --> look at sediemtn


Urien gram stain and culture --> infectious hematuria
Cytologic eval --> malignancy suspected
Renal imaging via CT or U/S --> visualize the renal parenchyma and vascular structures
Cystoscopy for the bladder

Kidney trauma
Renal stones and crystals
Glomerulonephritis
Infection (pyelo)
Neoplasia (RCC)
Vascular injury (vasculitis, renal thrombosis)

Causes of extrarenal hematuria

Trauma (foley!!)
Infections (urethritis, prostatitis, cystitis)
Nephrolithiasis (ureteral stones)
Neoplasia (prostate, bladder)

Nephrotic syndrome, four features besides


being noninflammatory

Edema
Hypoalbum
Hyperlipidemia
Proteinuria >3g/day

Nephritic syndrome

Hematuria
Edema
HTN
Low degree of proteinuria (<1-2 g/day)

Causes of vasculitis-related GN

Wegener
Churg-strauss
PAN
MPA
HSP

Infection related GN?

PIGN, PSGN
or Infective endocarditis related
glomerulonephritis

How to distinguish between IgA


nephropathy and PSGN

PSGN usually occurs way after throat infection has ended...


IgA nephropathy may occur concurrently'
Also, PSGN has hypocomplement, ASO titers. IgA will also have mesangial IgA
on biopsy.

Heat exhaustion may lead to what kidney


findigns

Goodpasture vs Wegeners

Rhabdomyolysis and relase of myoglobin

Goodpasture --> young males, hemoptysis and hematuria. Ab


against IV collagen
Wegener --> older adults, includes more systemic symptoms such
as arthralgias, myalgias, and sinonsasl symptoms. cANCA+

Most common causes of nephrotic


syndrome

Definition of nephrotic syndrome

1/3 --> systemic disease like DM or lupus


The rest have primaryrenal disease, with one of four:
- Minimal change disease
- Membranous nephropathy
- Focal segmental glomerulosclerosis
- Membranoproliferative GN

Urine protein excretion > 3.5 g / 24 h


Serum hypoalbuminemia
Hyperlipidemia
Edema

Microalbumin definition

30-300mg

Urinalysis in nephrotic syndrome

Protein
Waxy casts
Oval fat bodies --> Maltese crosses under polarized
light, only if hyperlipidemia is present

How does nephrotic syndrome typically


present

Edema
U/A proteinuria

Workup for newly presenting nephrotic


syndrome

Serum glucose and HgbA1c


ANA
MM workup amyloidosis
Viral serologies: HIV, viral hepatitis
Less common causes: various cnacers, meds such as NSAIDs, heavy metals such as mercury, heredtiary renal conditions
Diabetes is the most common!!

Treatment of nephrotic syndrome

Consequences of renal protein wasting in


nephrotic syndrome

Treatment of underlying disease


Salt restriction for edema
Diuretics: which are mostly highly protein bound, so very large
doses may be required.
Protein-restricted diet
Decreased ATIII and Protein C&S ---> hypercoag
Hypogamma-glob --> increased infection risk, esp pneumo
Fe anemia caused by hypotransferrrin
Vit d deficiency b/c of loss of Vit-D binding protein

Time period between appearance of


microalbuminuria and overt ESRD

5-15 years

Life expectancy once someone on DMESRD needs to be put on dialysis

2 years

Things that delay progression of DMrelated renal disease

Tight glycemic control 6.5-7 (only during micro stage, not for more advanced).
Strict BP control (130/80 less than), esp in >1g/d
ACEI or ARB --> reduces progression of renal disease independnet of BP
control
Cardiovascualr risk reduction reduces overall mortality: LDL<100,

Acute presentation of knee effusion, limited ROM,


and signs of inflammation (fever, erythema, warmth,
tenderness) in the setting of no trauma

Septic arthritis (may be gonococcal if at risk)


Crystalline arthritis (gout or pseudogout)

Drugs that trigger gout

Thiazides, decrease uric acid secretion.


EtOH, increase uric acid production

Joints affected in gout

First MTP (Podagra)


Ankle
Midfoot
Knee

Pseudogout joints

Large joints, e.g. the knee.


Wrist or MTP as well...

Arthritis of gonococcal disease

Nongonococcal septic arthritis, what joints

Basic approach in approaching a sore joint

Migratory arthralgias and tenosynovitis, often involving the hands and wrists
Usually assoc with pustular lesions, before progressing to a purulent monoarthritis or
oligoarthritis.

Knee
Hip
Large joints! shoulder
Differentiate arthritis from inflammatory conditions adjacent tot the joint,
such as cellulitis or bursitis. You can do so through ROM, which should be
limited in painful everywhere during arthritis...
Joint movement hat is nto limtied by passive motion suggests bursitis rather
than arthritis...

Septic arthritis leukcoytes vs inflammaroy


or noninflammatory leukocytes

>100000 septic
Inflammatory 2000-75000
Noninflammatory - 50-1000
<200 is normal..

Anything over >2000 WBC should be considered inflammatory and thus infected until proven otherwise.

Glucose in septic arthritis

Gout vs pseudogout crystals

<50mg/dl lower than blood, same goes for


inflammatory arthritis.

Gout: Monosodium urate crystals, which are needle shaped, negatively


birefringent (appearing yellow), typically within a PMN
Pseudogout: Calcium pyrophosphate dehydrate crystals. Short, rhomboid,
weakly positive birefringent (appearing blue). Even if crystals are seen,
infection must be excluded!!!

If crystals are seen in joint aspirate, does


this point conclusively to gout or
pseudogout?

NO. Crystals and infection may coexist in the same


joint, and chronic arthritis or previous joint damage,
such as occurs in gout, may predispose that joint to
infection (hematogenous).

Septic arthritis, how often is the gram


stain and culture positive?

60-80% of cases... False negative prior antibiotic use, or fastidious


organisms

Joint aspirates in gonoccocal arhtritis

Typically negative!!! But cultures of blood or pustular lesions are


typically positive... Or may be made from another site like
urethritis....

Synovial biopsy good for what

Usually necessary to diagnose arthritis


caused by tuberculosis or
hemochromatosis....

What suggests pseudogout on XR

Chondrocalcinosis aka linear calcium deposition...

Causes of nongonococcal septic arthritis

Usually S aureus.... Treamtent would involve antistaph penicillin such as nafcillin or Vanc if
MRSA is suspected....

Treatment of nongonococcal septic


arthritis

When is open surgical drainage or


arthroscopy required?

Gout four stagse?

Drain purulent joint fluid (usually by repeated percutaneous aspiration)


Antibiotics to treat SA or MRSA
SA: Nafcillin etc.
MRSA: Vancomycin

Joint fluid is loculated, or when shoulders, hips, or


sacroiliac joints are involved....

ASymptomatic hyperuricemia
Acute gouty arthritis --> monoarticular pain, occuring at night
Intercritical gout (period between attacks)
Chronic tophaceous gout,with intercritical periods no longer
symptomatic.

Sign of chronic gout

Subcutaneous tophaceous deposits of monosodium


urate.

Does lowering urate level prevent


development of gout?

No.

Mainstay of therapy for gout

Indomethacin (potent NSAID)


Colchicine TID until joint symptoms abate,
but dosing is limited by GI side effects N/D

Gout in the presence of renal


insufficinecy?

INdomethacin and Colchicine may be CI


Intraarticular glucocorticoid injection or oral steroids
may help instead...

Should you initiate uric acid lowering


treatment for an acute attack?

NO/. Any sudden increase OR decrease in


urate levels may precipitate further
attacks.

When is lowering uric acid appropriate in


gout?

During intercritical periods


<6.0 mg/dl

Dietary restrictions in gout

Avoid organrich foods such as the liver.... and avoiding alcohol.

Drugs that lower uric acid

Probenecid: increase uric acid secretion. (ineffective in renal failure)


Allopurinol
Febuxostat
Allopurinol requires dose adjustment in renal disease, but Febuxostat does not.

Pseudogout treatment

Acute attakcs same as gout: NSAIDs, Colchicine, systemic or intraarticular


steroids...
Prophylaxis with colchicine may be helpful, but no way to prevent crystal
formation...

Distinctive features of RA

Chronic, symmetric, erosive synovitis of peripheral joints. If


untreated, leads to deformity and destruction of joints due to
erosion of cartilage and bone.

RA diagnosis is what

Clinical one... based on clinical findings,


lab abnormalities, radiographic erosions.

Pain + without inflammatory changes of a


joint, think what?

OA
Fibromyalgia
Hypothyroidism
Neuropathic pain
Depression

What suggests joint pain of inflammatory


problems?

Soft tissue swelling


Joint effusion
Tenderness
Warmth of the joint
Limitation of both active and passive ROM

Swelling + limitation of only active motion


and not passive motion?

Extraarticular inflammation such as bursitis or


tendonitis.

Polyarticular inflammatory disease, what


is the next step in work up?

Chronicitiy
<6 weeks --> acute, major considerations are viral
infection, or earliest manifestations of true rheumatic
disease.
HBV
HCV
Rubella
Parvovirus B19

Causes o fviral arthritis

Treatment: symptoms relief with NSAIDs

Most characteristic feature of RA among


polyarticular inflammatory chronic
arthritis
Rheumatic fever quality of arthritis

In what syndromes do you get "sausage


digits," and why

Definite diagnosis of psoriatic arthritis


cannot be made until what happens

Reactive arthritis vs Reiter syndrome

Symmetric!!!
Other autoimmune rheumatic diseases such as SLE and psoriatic arthritis, are often
asymmetric.... Lupus may be symmetric but is also characterized by other symptoms such as:
malar rash, serositis (pleuritis, psoriatic arthritis), renal diseas with protein or blood, CNS
manifestations, hemolytic anemia leukopenia, lymphopenia,or thrombocytopenia.

Symmetric polyarthritis, with an acute


febrile illness lasting 6-8weeks

Psoriatic arthritis
Reactive arthritis
This is due to inflammation that is not limited to the joints, but also occurs at
the periosteum, along tnedons, and at insertion points along the bone....

Without evidence of skin or nail changes typical of psoriasis (nail pitting, scaly
plaques). Arthritis can precede rash or vice versa...

Reactive arthritis is an asymmetric inflammatory arthritis that follows infection of GI tract or GU tract --> Salmonella,
Shigella, Campylobacter, Yersinia, or Chlamydia....
Reiter syndrome --> triad of arthritis, uveitis, urethritis...

RA typically involves what joints

MCP, wrists, PIP... DIPs are usually spared.

OA typically involves what joints

DIP... Heberden nodes, contrast from


Bouchard nodes.

Stiffness in the morning >1hr

Any inflammatory condition that reflects severity of joint


inflammation.

Where are Rheumatoid nodules found,


and in how many RA patients do they
occur
What is RF

Utilitay of RF titers

Subcutaneous nodules typicallhy found over extensor surfaces --> proximal ulna or other pressure points.
They only occur in 20-30% of patients with RA, but are highly specific for RA.

Ig that react to Fc portion of IgG... They are usually IgMs, which are found in 80-85% of patients
with RA.

Not specific for RA (5% of people healthy can have them)


Found in 80-85% of patients with RA...
High RF titers have prognostic utility for more severe systemic and
progressive disease.

Better biomarker than RF

Radiologic findings in RA

Anti-cyclic citrullinated peptides... have same sensitivity as RF but are highly


specific (95%).. Anti-CCP also portends worse outcomes.

Erosion of periarticular bone and cartilage destruction with loss of joint space --> may help diagnosis...
Joint space narrowing
Subchondral polysclerosis
Marginal osteophyte formation
Cyst formation
Typical XR findings do not develop unti later in disease process, and diagnosis is usually made already.

Extraarticular manifestations of RA

Heme manifestations of RA

Felty syndrome

Vasculitic syndrome with ischemic ulcers


Ocular manifestations --> Keratoconjunctivitis sicca (Sjogren)
Interstitial lung disease
Cardiac manifestations
Severe neuro stuff with myelopathy related to cervical spine instability...
Entrapment neuropathy --> carpal tunnel

ACD

Most common with nodule forming RA, this is a combination of:


- RA
- Splenomegaly
- Leukopenia
- LAD
- Thrombocytopenia

DMARDs stand for what

Nonbiologic DMARDs

Disease-modifying antirheumatic drugs,


may be biologic or nonbiologic

MTX
Hydroxychloroquine
Sulfasalazine
Minocycline
Leflunomide
MTX ois often used first, rapid onset of action, and higher tolerability and patient compliance.

TNF antagonists

Etanercept
Infliximab
Adalimumab
Reduces disease severity with RA in weeks.

Other biologics besides TNF antagonists

IL-1 receptor antagonists: Abatacept, which is a soluble fusion


protein of IgG and CTLA-4
Rituximab: chimeric monoclonal antibody against CD20, a cell
surface molecule of B-lymphocytes

Crazy DMARDS

Hydroxychloroquine
Sulfasalazine
Oral and parenteral gold
Penicillamine

Alcoholic ketoacidosis, best initial


treatment

D5 with NS

Alcoholic ketoacidosis, what to rule out

Methanol
Ethylene glycol
Would present with similar labs, but may be more serious or even fatal

Anion gap calculation


Urine anion gap

NA - cl - hco3
Na+K - Cl --> this is an estimate of NH4+ excretion

Normal systemic pH

7.35-7.45

Normal anion gap

10-12
This will be lower if the patients are hypoalbuminemic, or if there are high levels of immunoglobulins as in
MM

Lactic acidosis most commonly occurs


when?

Acute illness --> poor tissue perfusion, such as septic shock, heart failure,
severe anemia, or poisoning affecting tissue oxygen delivery or cellular
respiration (CO, cyanide).
Treatment is aimed at correcting the underlying condition... NaHCO3 may be
used in severe acidemia

Severe acidemia requiring bicarb

Causes of anion gap metabolic acidosis

Ketoacids accumulate in DKA

<7.1

Lactic acidosis
Ketoacidosis (DAS, or diabetic, alcoholic, starvation)
Toxins (ethylene glycol, methanol, salicylates, propylene glycol)
Renal failure (acute and chronic)

Acetoacetate
Beta-hydroxybutyrate

Pathogenesis of alcoholic ketoacdiosis

Decreased carb intake reduces inulin secretion.


Alcohol-induced inhibition of gluconeogenesis --> stimulation of
lipolysis and contributes to increased ketoacid formation,
predominantly beta-hydroxybutyrate

Why might the Acetest only be weakly


positive in alcoholic ketoacidosis vs DKA

Test onlhy really picks up acetoacetate, but not betahydroxybutyrate (the predoimnant ketone
produced in AKA). This can lead to underestimation of the degree of ketosis.
This leads to a funny thing when the patient is treated., bc as they are the foration of acetoacetate
is favored, so the degree of measured ketosis may appear to paradoxically worsen.

Ketoacidosis but with normal, low, or


somewhat elevated glucose?

Think alcoholic ketoacidosis

What should be added to any D5 fluid


given to a alcoholic

Thiamine 100mg

Methanol, why is it toxic

Metabolized to formaldehyde and formic acid


by alcoholideH --> optic nerve and CNS
injury.

Ethylene glycol, why is it toxic

Metabolized by ADH to glycolate, glyoxylate, and oxalate --> acute renal failure due to glycolate
induced damage to tubules.. tubular obstruction from precipitated oxalate crystals.

Metabolic gap acidosis plus high osmolal


gap?

Serum osm calculation

Methanol
Ethylene glycol
These things cannot be normally detected, but can be inferred with the above
constellation of lab findings.

2*Na + glucose/18 + BUN/2.8


Measured - calculated osm, if hte gap is greaterthan 10 --> presence of
another solute is suspected.

Normal osmolal gap

>10

Additional finding in ethylene glycol


poisoning

Oxalate crystals in the urine

Treatment of both methanol and ethylene


glycol ingestion

Fomepizole, an ADH inhibitor, reduces


formation of toxic metabolites.

Main cause of non-AG metabolic acidosis

Bicarb loss from GI tract or kidneys... you get a rise in chloride concentration that approximates
fall in bicarb concentration (hyperchloremic metabolic acidosis)

Urine in non-aG metabolic acidosis

Diarrhea and hypokalemia leads to renal


syntehsis and secretion of ammonia, and
thus pH of urine is >5.5

Acidosis with high urine pH, how to


differentiate non-anion gap metabolic
acidosis from RTA....
Main NAGMA causes

Basis of urine NH4, which cannot directly be measured but can be estimated with UAG
NH4 excretion is low in RTA, but high in NAGMA
UAG = Na +K - Cl. When this is negative, there is high NH4+ to balance out excess Cl.... thus an extrarenal cause of
acidosis... GI probably...
If UAG is positive, this indicates that there is impaired NH4+ excretion, and thus distal Type 1 RTA, hypoaldosteronism, or
Type 4 RTA.. Can also happen in advanced chronic kidney disease.

GI bicarb loss (diarrhea, pancreatic, small bowel drainage, ureterosigmoidostomy, jejunal loop, ileal loop)
Renal acidosis (hypokalemia RTA-II, RTA-I; Hyperkalemia Type IV RTA due to mineralocorticoid deficiency
or resistance).
Druginduced
Other causes: Acid loads, expansion acidosis (due to rapid saline administration)

Drug induced causes of NAGMA

Those that induce hyperkalemia: Potassium sparing diuretics


(amiloride, triamterene, spironolactone)
Trimethoprim
ACEI and ARBs
NSAIDs

Other causes of NAGMA

Acid loads like through hyperalimentation


Expansion acidosis.

Metabolic alkalosis how

Loss of acid
Excess endogenous production of bicarb (bicarb normally created in the
kidneys, but may fail to be expressed). This happens in Hyperaldosteronism, or
ECF volume contraction and hypokalemia due to variosu causes (vomiting, NG
suction, diretics)..

Metabolic alkalosses can be divided into


what two categories

"Chloride resistant" - cannot be corrected by adminstration of sodium


chloride, as in primary aldosteronism, RAS, Cushings. UCl >20mEq/L

What RTA is commonly seen in patients


with diabetic nephropathy?

Type 4 RTA due to generalized distal nephron dysfunction... Low


plasma renin activity is common in diabetic patients, leading to
hyporeninemic hypoaldosteronisms... Hyperkalemia (unlike other
RTAs), and amilde hyperchloremic metabolic acidosis.

What is useful for judging the volume


status of a patient with metabolic alkalosis

Urine chloride, used to classify if a

Formic acid effects on the body

Mental status depression


Papilledema
Optic neuritis
Metabolic acidosis

Respiratory alkalosis symptoms

Symptoms of cerebral vasoconstriction (dizzines) and transient


hypocalcemia (perioral numbness and paresthesias)

What lab factor can indicate that a


metabolic alkalosis will respond to saline
infusion

Low urine chloride

When can imaging studies of


musculoskeletal back pain be considered

Usually resolves in 6 minutes. If not...

Cauda equina syndrome

"Chloride responsive" as in vomiting, NG section, diureticss. UCl <10


mEq/L

Lower back pain


Saddle anesthesia
Bowel or bladder dysfunction with possible lower extremity weakness and loss
of reflexes caused by compression of multiple sacral nerve roots...
Surgical emergency

Sciatica

Pain in distribution of lumbar or sacral


nerve roots, w/ or w/o motor or sensory
deficits

Spondylolisthesis

Anterior displacement of upper vertebral body on the lower body -->


symptoms and signs of spinal stenosis...

Spondylolysis

Defect in the pars interarticularis (scotty dog), either


congenital or secondary to a stress fracture, leading
to LBP, back muscle spasms, or no symptoms.

Back pain is secondary to what

URIs
Millions of dollars are expended on this
problem each year.

LBP, problems to exclude

Malignancy
Infection
Dangerous neurologic processes: spinal cord
compression, cauda equina sydnrome.

If patients do not improve in back pain in 4


weeks, what should happen?

Undergo further evaluation for systemic or rheumatic disease, and


to clarify the anatomic cause. Esp in patients with localized pain,
nocturnal pain, or sciatica.

Pain that radiates down back of leg, what


nerve roots?

Herniated intervertebral disc at the L4-L5, L5-S1 level. Patients


typically report aching pain in the buttock and paresthesias
radiating into posterior thigh, calf, lateral foreleg... radiation to
below the knee is more likely to indicat ea true radiculopathy rather
than just extension to the posterior thigh.

Back pain may be due to visceral pain from


what structures

Pelvic disease
Renal disease
Aortic aneurysm
GI disease

Nonmechanical causes of lower back pain

Neoplasia
Infection
Inflammatory arhtritis
Paget disease
All account for 1% of total LBP

Breakdown of musculoskeletal back pain,


which total accounts for 97% total LBP

Lumbar sprain and strain - 70%


DJD - 10%
Hernaited disk - 4%
Spinal stenosis - 3%
Trauma - 1%
Congenital disease, e.g. kyphoscoliosis - <1%

"Red flag" symptoms for back pain

New onset of pain <20 or >50


Fever
Unintentional weight loss
Severe nighttime pain or pain that is worse in the supine position
Bowel or bladder incontinence
History of cancer
Immunosuppression
Saddle anesthesia
Major motor weakness..

Cancers that met to the spine

Lung
Breast
Prostate
Lymphoma
GI tumors
Malignant Melanoma
MM --> bone pain, renal failure, anemia

Given worrisome signs, good initial work


up

AP and Lat
Sedimentation rate
CBC
MRI may be reserved because it is usually not required to make a diagnosis.

Tenderness ove rht e back, different


sources?

Over spinal processes --> destructive lesion of the back itself


Musculoskeletal --> tenderness inthe muscular paraspinal area.

How to distinguish pain emanating from


the sacroiliac joint

Patrick maneuver --> making a figure four out of the


legs..

Bed rest vs activity

Activity --> always activity

TB osteomyelitis of the spine aka

Pott disease

Hgb levels in men and women to indicate


anemia

<13 men
<12 women

MCV micro, normo, macro

Micro<80
Normo80-100
Macro>100

How old is a reticulocyte

1-1.5 days old

Normal reticulocyte count

1%

Normal daily intake of iron

15mg, but 1-2mg is absorbed.

Most frequent cause of iron deficiency


anemia in men

Chronic GI tract occult bleeding

Parasitic cause of Fe deficiency

Hookworm

Causes of Fe malabsorption

CELIAC DISEASE
Gastrectomy
IBD e.g. CD

When might inadequate intake of iron take


place

Infancy/adolescence
Pregnancy
Vegetarian diet

Fe deficiency: ferritin and TIBC

Ferritin levels will fall


TIBC will increase... (total iron binding
capacity)
Fatigue
Shortness of breath
Dizziness
Headache
Palpitations
Impaired concentration

Typical symptoms of anemia

Plummer Vinsion syndrome

Dysphagia associated with postcricoid esophageal web --> Fe deficiency


Postmenopausal women
Presents with dysphagia, esophageal webs, iron deficiency anemia.May complain of burning
sensation with tongue and oral mucosa, atrophy of lingual papillae --> smooth, shiny red dorsum
of the tongue.

Physical signs and smptosm of chronic


severe iron deficiency

Pica or ice cravings (pagophagia)


Glossitis
Cheilosis
Koilonychia
Postcricoid esophageal web --> Plummer-Vinson syndrome

RDW in microcytic anemia

Increased: Fe deficiency
Decreased: uniform sizes.. may suggest chronic
disease such as thalassemia, or even Fe deficiency
with concomitant anemia of chronic disease...

Causes of microcytic anemia

Iron deficiency
Thalassemia
Sideroblastic anemia
Lead poisoning

Causes of normocytic anemia

Acute blood loss


Hemolysis
ACD
Anemia of renal failure
Myelodysplastic syndromes

Macrocytic anemai causes

Folate / B12 deficiency


Drug toxicity: Zidovudine
Alcoholism / chronic liver disease

What single lab value is a reliable indicator


for iron deficiency?

Low serum ferritin... Specific, but may not be


sensitive because this may rise as an acute reactant in
response to inflammation, hepatitis, RA, hodgkin
disease, etc.

Sideroblastic anemia

Bone marrow produces abnormal RBCs --> microcytic and


hypochromic.

Iron studies in sideroblastic anemia

Increases in serum iron, and serum ferritin, and


saturation of transferrin.

Pathognomonic signs of sideroblastic


anemia

Stippled RBCs in the peripheral blood smear.


Iron stain in bone marrow --> engorged mitochondria in the
developing RBCs which are called ringed sideroblasts

Treatment of Fe deficiency anemia

Oral ferrous sulfate 325mg BID or TID. Correction usually


occurs within 6 weeks. Therapy to continue for at least 6
months to replenish iron stores.

Side effects of iron pills

GI: constipation, nausea, abdominal cramping. Taking iron with meals can help with this but
may decrease absorption.

IV iron when?

Poor absorption state (Celiac, CKD). Or with


excessive intolerance to oral therapy.

Iron dextran IV, problems?

Anaphylaxis. Less of a problem with newer parenteral


IV

Most common cause of hematochezia in


patients older than 60?

Diverticular hemorrhage --> typically presents as passage painless


of bright red blood. Only 20% of those with diverticulosis will
present with blood. Generally hemorrhage is abrupt in onset and
abrupt in resolution.

Of those with diverticulums, who gets


diverticulitis

20%

Diverticulitis, uncomplicated vs
complicated

75% vs 25%

Complicatd diverticulitis

Uncomplicated: abdominla pain, fever leukocytosis, anorexia, constipation, obstipation

Abscess
Perforation
Stricture
Fistula
The most common is an abscess.

Preferred mdoality for diagnsing


diverticulitis

How to manage uncomplicated


diverticulitis

Fistula formation in diverticulitis

Complications of diverticulitis

CT scan --> sigmoid diverticula, thickening of the bowel wall to more than 4mm, pericolic fat stranding signifying
inflammation, or the finding of a diverticular abscess.

Bowel rest
Antibiotics --> oral quinolone plus metronidazole,
or amoxicillin-clavulanate for 10-14 days.
Majority is colovesical with male predominance (uterus protects bladder in females)..
Others include colovaginal, coloenteric, colouterine, coloureteral..
Colocutaneous is extremely rare...

Abscess
Fistulas
Obstruction
Strictures

Indications for surgical management of


diverticulitis

Generalized peritonitis
Uncontrolled sepsis
Perforation
Clinical deterioration

Most common cause of an acute abdomen


at any age

Appendicits, no matter the side

Nadir of WBC count after initiation chemo

Occurs 7-14 days after onset

Definition of neutropenia

ANC<500cells/mm3

Most common sources of infection for


neutropenic fever

Gram positives from the skin or oral cavity


Gram negative bacteria from the bowel

Definition of fever

Single temp >= 101 (38.3)


Temp for an hour >= 100.4 for 1 hour or more.

What is mucositis

Breakdown of skin and mucosal barriers as a result of


chemotherapy or radiation. Mucositis can result in
bacteremia or fungemia...

Prevalence of neutropenic fever

5-10% of cancer patients will die of neutropenia-assoc infection.

Trends in neutropenic causes

Previoulsy gram neg, but increasingly gram positive


due to use of central venous catheters.
Any sign of skin breakdown --> vanc

What if neutropenic patient is febrile even


after antibacterial therapy

Empiric antifungal therapy with either fluconazole or amphotericin B.

Most common organisms causing line


infections

S. epiermidis
S. aureus

Preventative measures for neutropenic


patients

Immunized: Pneumo and infleunza


G-CSF --> aka Filgrastim, used to shortern
duration and depth of neutropenia.

Acute chest syndrome, how does it prsent

CoagStaph --> usually you can keep the line in place. S. aureus or
gram negs, or fungemia, you need to remove the catheter.

Chest pain and tachypnea


Can result from infectious or noninfectious processes.
Usually presents with soem combo of chest pain, fever, hypoxia, and new pulm
infiltrate on CXR... often AChestS and pneumonia cannot be distinguished
initially.

Treat Acute Chest Syndrome?

Antibiotics
Gram staina nd culture of sputum
Admission
These patients need oxygen, IVF, and analgesia

Acute Chest Syndrome characterized by

Fever
Tachycardia
Chest pain
Leukocytosis
Pulmonary infiltrates

What promotes sickling

Hypoxia
Acidosis
Dehydration
Variations in body temperature

Prevalence of sickle cell gene

8% of AA are SCT, with one in 625 affected by the disease.

Most common sites for acute pain


episodes

Long bones of arms and legs


Vertebral column
Sternum
Acute pain episodes are precipitated by infection, hypoxia, cold exposure,
dehydration, venous stasis, or acidosis... usually they last 2-7 days.

Causes of acute chest syndrome

Aplastic crisis

May be causd by infection, or not... intrapulm sickling itself could be a


primary event. But you cannot distinguish this clinically, thus empiric
antibiotic therapy is used.

Viral suppresion of RBC precursors most


often by parvovirus B19.

Acute chest syndrome treatment

Oxygen
Analgesia
Antibiotics

HgbF stimulators

Hydroxyurea
Decitabine

Causes of acute chest syndrome

Pneumonia
Vasoocclusion
Pulmonary embolism

Cerebritis

aka abscess, or a focal brain parenchymal


infection

Most common meningitis bacterial isolate

S. pneumoniae
N. meningitidis is a close second
GBS - 10% of cases

Who gets GBS meningitis

Noenates
Patients >50 or with chronic illness such as diabetes or liver
disease.

Who gets listeria monocytogenes meingitis

Pregnant women, elderly, or patients with cellmediated immunity impariemtn such as AIDS

S. pneumoniae now resistant to what

PCN
Cephalosporins

Meningitis in patients following surgical


procedures such as BP shunt placement

S. aureus
S. epidermidis

How many meningitis patients get


confusion or altered level of
consciousness?

75%

Viral cause of meningitis

Enteroviruses, most common in summer and fall. Headache PLUS


gastroenteritis!!!

Meningitis cause in HIV

Cryptococcus

LP within how long of antibiotics

Preferably within 30 minutes


Up to two hours probably okay, as it will not
significantly alter CSF protein, glucose, or WBC
count.

HSV meningitis

MRI --> enhancement of temporal lobes

Purpuric skin lesiosn + meningitis

You can biopsy these and they will be + for


N. meningitidis.

Empiric therapy for meningitis

High dose third gen cephalosporin


+ Vancomycin
Ampicillin? For Liosteria

Steroids in meingitis

Controversial... One studyshows decreased


mortality.

Close contacts for meningococcemia

Rifampin BID x 2 days


Ciprofloxacin single dose.

Listeria type of organism

Gram positive rod


Causes 10% of all cases of meningitis.
More common in the elderly, and in other patients with impaired cell mediated immunity-->
chemo
More common in neonates as well..
NOT senstiive to cephalosporins.It is to ampicillin.

CT before LP when?

Risk of brain herniation high: new onset seizures,


signs suspicious for space occuying lesions such as
papilleedema and focal neuro signs

Tricuspid endocarditis murmur

Janeway lesions vs Osler nodes

Harsh holosystolic murmur over LLSB,


increases with inspiration.

Janeway lesions are painless hemorrhagic macules on palms and soles, septic emboli possibly
Osler nodes - painful, palpable erythematous lesions --> pads of fingers and toes, they represent vasculitic lesions caused by
immune complexes.

Roth spots

Hemorrhagic retinal lesions with white ceenters, due to infectious


endocarditis. Immune complex mediated?

Fever in endocarditis?

95%

Subacute endocarditis presentation

Constitutional symptoms such as anorexia, weight loss, night


sweats, and findings attributable to immune complex deposition -> Oslers, Janeway, and Roth.. only seenin 20-25% of cases..

Acute vs subacute blood cultures

Acute three over 2-3 hours


Subacute 3 over 24 hours.

Older pateintw tih GU tract disease or


instrumentation, endocarditis

Enterococci

Who gets Strep bovis endocarditis

Elderly patients often with underlying mucosal


lesions such as an adenoma or malignancy.

CAndida endocarditis

Intravascular catheters
IVDU

HACEK organisms

Haemophilus aphrophilus
Actinobacillus
Cardiobacterium hominis
Eikenella corrodens
Kingella kingae

Stroke +febrile?

Endocarditis!! the heart!!

S aureus endocarditis

Nafcilllin + Gent for synergy initially to


help resolve bactermeia.

Most strep speicies endocarditis

PCN G

When is Vanc used for endocaridtis

MRSA
or
Coagulase-negative staph

HACEK endocarditis regimen

Ceftriaxone

Indications for surgical management of


endocarditis

Intractable CHF, >1 serious systemic embolic episodes, or large (>10mm) veg with high risk for embolism
Uncontrolled infection, e.g. positive cultures after 7 days of therapy
No effective antimicrobial therapy (fungal)
Most cases of prosthetic valve endocarditis
Local suppurative complications e.g. myocardial abscess

Drug of choice for antibiotic prophylaxis

Amoxicillin
Used for previous infective endo, prosthetic herat valves, CHD or
reparired CHD, valve regurg caused by structurally abnormal valve
in cardiac transplant patients.

Culture neg endocarditis causes

antibiotics
infection with fungi
HACEK group

How much fluid can cause a tamponade

Depends on how quickly the fluid


develops. As little as 200ml in certain
circumstances.

Triad of tamponade

Beck triad --> Hypotension, elevated JVP, small/quiet heart -->


Acute tamponade

Chronic tamponade

May look more like CHF --> cardiomegaly on CXR (although there should be no pulm edema),
dyspena, elevated JVP, hepatomegaly, and peripheral edema... A high index of suspicion is
required..

Tamponade should be considered when

Anyone with elevated JVP and


hypotension.

Most important physical signs to look for


in tamponade?

Pulsus paradoxus --> Drop in SBP >10mmHg during inspiration...


which is just an exaggeration of the normal physiologic response.
If severe, can feel pulse variation at the radial.

Who gets pulsus paradoxus

Most concerning for tamponade


Can also be seen in COPD --> defects of intrathoracic pressure
during inspiration.

Constrictive pericarditis

thickened fibrotic sac --> impairs diastolic filling...


due to ganulation tissue formation... Typically a
complication of previous pericarditsi --> acute or
chronic.

Most common causes of constrictive


pericarditsi

Historically TB
currently, radiation therapy, cardiac surgery, or any cause of acute pericarditis such as viral
infection, uremia, or malignancy.

Constrictive pericarditis presentation

Pathophys is similar to that of tamponade. Restricted ability of the ventricles to fill during
diastole b/c of thickened noncompliant pericardium.

Chronic --> chronic and slowly progressive weakness and fatigue


and exertional dyspnea... chronic LE edema, HM, ascites.. Elevated
JVP, but pulsus paradoxus is typically ABSENT!
Kussmaul sign is present --> increase in JVP during inspiration.

Kussmaul sign

Physical findings of constrictive


pericarditis
Imaging?
Restrictive cardiomyopathy causes

Increase in JVP during inspiration. This is paradoxical because


JVP should fall during inspiration. Normally blood get sucked into
the right side of the heart, but this is restricted, so blood gets sucked
into the jugular vein instead.

Kussmaul sign (elevated JVP during inspiration)


Pericardial knock - high pitched early diastolic sound occurring just after
aortic valve closure.
CXR - calcified pericardium

Not very common in western world


Amyloidosis, usually in the elderly
Fibrosis following radiation therapy or open heart surgery
Africa? Much more common because of a process called endomyocardial fibrosis, which is
fibrosis of the endocardium along wit hfever and marked eosinophilia accounting for up to 25%
of deaths due to heart disease.

Acute pericarditis clinical features

Chest pain (positional!)


Fever
Pericardial rub

Restrictive cardiomyopathy clinical


featuers

No PP or Kussmauls
Progressive exertional dyspnea and dependnet edema.

Endomyocardial fibrsis

Cause of Restrictive cardiomyopathy common in


developing countries, associated with eosinophilia....

Causes of DCM

Viral myocarditis
Beriberi (wet)
Doxorubicin deficiency

Chancre of primary syphilis

Shallow clean ulcer without exudates or


erythema, nontender
Assoc with LAD

Treatment of primary syphilis

Single IM injection of benzathine PCN-G

Common causes of genital ulcers

Syphilis (nontender)
Chancroid (ducreyi)
Herpes
Superficially ifnected skin lesion.

Seconday syphilis

Disseminated infection manifesting in a pruritic, maculopapular diffuse


rash that classically involves the palms and soles, or the flat moist
lesion of condyloma lata.
Or also lesions orally --> mucous patches...

Tertiary syphilis

Chancre vs chancroid

Gummas of the CNS, cardiovascular system, or skin and SQ tissues.

Chancre - Non erythematous with rolled borders, clean base, very firm,
nontender. Can hav LAD
Chancroid - Painful, exudative, ragged borders, necrotic base, bleeds
easily. LAD is suppurative..

Condyloma lata

Mediator of late stage syphilis

Wide warts... gray papillomatous lesion


found in intertriginous areas and patchy
hair loss.
immune reaction to disseminated organism --> Proliferative obliterative
endarteritis... In some organs this can organize into granulomas with an
amorphous, coagulated center called a gumma. Skin,liver, bone, CNS, etc.
In aorta --> obliterative endarteritis can involve the vasa vasorum whcih leads
to necrosis of the media of the arterial wall. Aorta aneurysms (saccular)

Neurosyphilis

May cause vasculitis --> ischemia, stroke, and focal neuro deficits.
Personality changes or dementia
Demyelination of the posterior column with wide-based gait, loss of
proprioception (Tabes dorsalis). Or CN involvement --> Argyll
Robertson pupil

Argyll Robertson pupil

Pupil that accomodates but does not react to light


Sign of neurosyphilis.

Nonspecific serology testing for syphilis

RPR
VDRL
These are tests for lipid antigens that occur as part of host reaction to T.
pallidum and are fairly sensitive.... But low titers may be nonspecific, may
result in false positives.

Confirmatory testing for syphilis

FTA-ABS
MHA-TP
Dark field microscopy can find T. pallidum fro ulcer scrapings...
this is classic but not really used today.

CNS involvement of syphilis on LP

+CSR VDRL or RPR in the setting of increased CSF leukocytes and


protein counts
Low glucose levels
False negs for VDRL in CSF are common.

Treatment of primary vs later stage


syphilis

Primary, Seconadry <1 year - one injection of benzathine PCN G (long lasting IM injection)
Late - Latent syphilis of unknown duration (presumed to be >1year) or with cardiovascula ro
gummas... Three weekly IM injections of benzathine PCN g
Tertiary syphilis or neurosyphilis - IV PCN 10-14 days.

Pregnant women allergic to PCN that have


syphilis, what is the treatment

How to gauge treatment response to


syphilis

Should be densesitized to PCN and treated, because this is the only treatment known to prevent congenital
infection...

FTA-ABS or MHA-TP are usually positive fo rlife


RPR serology will fall though.... A normal response is a fourfold drop in titers
w/i 3 months and a negative or near negative titer after 1 year. Suboptimal
response may mean inadequate treatment or undiagnosed tertiary disease.

Most common bacterial STD

Chlamydia, can be asymptomatic...


Cervicitis, urethritis (dysuria or UTI sx), and it may also cause pelvic
inflammatory disease (lower ab pain, vaginal discharge or dysuria, with fever
and systemic symptoms)... can lead to infertility due to tubal scarring.

Men with chlamydia

Urethritis (dysuria, urethral discharge)


Epididymitis (scrotal pain, fever)
Proctitis (rectal pain or diarrhea)

Treatment for Chlamydia

Single dose azithromycin


Course of doxycycline

Coinfection rate of GC and chlamydia

30%

Gonorrhea differences from chlamydia

Gonorrhea less likely to be asymptomatic


May cause disseminated infection: fever, migratory polyarthritis,
tenosynotivits of hands and feet, and a rash on the distal
extremities... These patients require hospitalization with IV
antibiotics, usually Ceftriaxone.

Primary syphliis lesion, how many will


have negative serology

1/3 --> can confirm via dark field microscopy or biopsy with special
stains...
Or just empiric treatment with benzathine PCN

CNS involvement of syphilis can only be


excluded via what

CSF testing.

COPD by PFT

FEV1<80% of predicted
FEV1/FVC <0.7 mild, 0.3-0.5 moderate, <0.3 severe
Reduced FEV1/FVC with minimal response to bronchodilators is the hallmark
of COPD.

A1AT deficiency presentation

By age 40 --> w/o cough or smoking history....


Therapy by A1AT enzyme replacement is available so
this is important to make this distinction.

Blue bloater vs pink puffer

Blue bloater - chronic bronchitis, overweight,


edematous, cyanotic
Pink puffer - emphysema, thin, ruddy cheeks

ABG in COPD

Normal in early disease


Then hypoxemic and hypercapnic
PaO2 50mmHg
PaCO2 50 mmHg
But near normalpH (50-50 club)
Acute exacerbation? More severe hypoxemia or hypercapnia,or respiratory acidosis noted on ABG --->
indication of impending respiratory failure and need for ventilatory support...

Extrapulmonary restrictive lung diseases

Poliomyelitis
Myasthenia gravis
Scoliosis

Intrapulmonary restrictive lung disease

Pneumonia
ARDS
Pulmonary edema
Interstitial fibrosis

Caution in giving patients oxygen in COPD

Signs of acute respiratory failure in COPD

Due to longstainding hypercapnia, there drive might be dependent


on "relative hypoxemia", so these people may become apneic if
excessive oxygen is adminsitered

Tachypnea (>40)
Inability to speak
ACcessory muscle usage with fatigue
Confusion
Restlessnes
Agitation
Lethargy
Rising PaCO2 level
Extreme hypoxemia
Treat with intubation wtih vent support to correct gas exchange.

Long term complications of COPD

Pulm hypertension
Seconadry erythrocytosis
Exercise limtiation
Impaired mental functioning
COPD who are stable? Smoking cessation

What alters natural history of disease in


cOPD

Supplemental oxygen
Lung volume reduction srugery

Who benefits from home O2 in copd

resting hypoxemia <PaO2 <55mmHg


SaO2<88%
This must be utiized at least 18h/day

PE findings consistenet with postnasal


drip

Chronically coughing nonsmoker with


negative CXR, top three diagnoses

Cobblestone appearance of the oropharynx (lymphoid hypderplasia)


Boggy erythematous nasal mucosa

Postnasal drip
Astma
GERD
Psychogenic cough is one of exclusion

Sinus radiograph features suggestive of


sinusitis

Opacification
Air fluid levels
Mucosal thickening

Challenge airway with what during PFT

Methacholine
Histamine

CXR findings in sarcoidosis

Hila LAD with or without interstitial infiltrates

AA female with fatigue, arthralgias, nagging dry


cough for six weeks, no SOB... Lugns clear
Bilateral pretibial tender erytehamtous raised
nodules

SARCOIDOSIS

Possible presentation of MM

Fatigue, absent mindedness, loss of appetite


an weight, nocutria,other weird things, pain.

Hypercalcemia syymptoms

Acute confusion
Fatigue
Lethargy

Hypercalcemia treatment

REstore voluem status, faciliate Ca excretion through IV saline.

Hypercalcemia of chronci duration with


PTH low and malig workup negative or not
suggestive of malignancy

Granulomatousdisease
FHH
Milk-alkali syndrome
Meds: Lithium, thiazides
Immobilization
Vit D or A toxicity
Adrenal insufficiency
Hyperthyroidism

Primary hyperparathyroidism presenation

Bones,groans,and psychiatric overtones....

diagnostic eval of primary


hyperparathyroidism

HyperCA
HypoP
Elevated PTH

Nephrolithiasis
Peptic ulcers
Mental status changes...

Sarcoidosis how high Ca2+

Excess 1,25(OH)2D synthesized in macrophages and lymphocytes.


Labs: low PTH, elevated 1,25(OH)2D
Elevated ACE level
Biopsy --> granulomas.

Renal insufficiency results in calcemia


how

Corrected calcium level

HyperCa in an otherwise healthy patient,


most common cause?

2/2 hyperparathyroidism as a result of partial


resistance to PTH effects
Bone pain, pruritus, ectopic calcification,
osteomalacia.

Add 0.8 Ca for every 1 albumin below 4....

Primary hyperparathyroidism... Most patients have no symptoms when Ca


<12, except perhaps some polyuria and dehydration... With levels more than 13
--> CNS symptoms (lethargy, stupor coma, mental status changes, psychosis),
GI syymptoms (Anorexa, nausea, constipation, PUD), Kidney problems
(polyuria, nephrolithiasis, prerenal azotemia), and musculoskeltal symptms
(arthalgias, myalgias, weakness)...

Hyperparathyroidism symptoms

Stones - kidneys
moans - abdominal pain
groans - myalgias
psychiatric overtones - mental status changes.

Acute onset of symptomatic


hypercalcemia, likely to have what?

Malignancy --> MM, lymphoma, leukemia. Soli dtumors: breast,


lung,kidney.

Mechanisms of malignancy assoc hyperCa

Bone lysis
Excess 1,25 vitamin D (lymphomas)
Parathyroid hormone-related protein (PTHrP)
In all cases, can distinguish from primary hyperpara by suppresssed PTH level.

Besides monoclonal antibody spike, other


ways to diagnosis MM

Therapy for symptomatic MM

Bisphosphonates, good for Calcium why?

>10% clonal plasma cells in the bone marrow


Lytic bone lesions

High dose pulsed dexamethasone


Combination chemotherapy with vincristine/doxorubicin or thalidomide.
Autologous stem cell transplant.

Inhibit osteoclast bone resorption

Most common inherited hypercoag states

ECG abnormalities in PE

Acute onset of hypoxemia in a ptient with


normal CXR

CXR findigns in PE if present

Factor V Leiden
Prothrombin gene mutations

Most common: sinus tachy


T wave inversion in the anterior leads
S1Q3T3 --> S wave in I, Q wave in III, Inverted T in lead III

PE until otherwise proven

Westermark sign - nonspecific prominence of central pulm artery w/


decreased pulm vascularity
Hampton hum p - peripheral wedge shaped density above diaphragm
Palla sign - enlargement of right descending PA

Who gets primary therapy for a PE

Right heart failure or hypoteniosn

Fondaparinux

Direct Factor Xa

Clubbing of fingers

Lung cancer
Chronic septic conditions --> bronchiectasis or lung abscess.

Definition of massive hemoptysis

100-600 ml of blood loss that is coughed within a 24h period.

Horner syndroem

Ptosis
Loss of pupillary dilation (miosis)
Anhydrosis on ipsilateral side.

Most common causes of hemoptysis in the


US

Bronchitis
Lung cancer
Prior eras:
TB
Lung abscess
Bronchiectasis
Most patients require CT scan, CXR.

Hemoptysis + acute onset of pleuritic


chest pain and dyspnea

ow much is lung cancer linked to smoking

Pulmonary embolism

85%
15% --> majority found in women for reasons that are unknown...
Thoracic radiation exposure, exposure to environmental toxins such as
absestos or radon

How many lung cancers are asymptomatic


when diagnosed

5-15%... In these cases a lung nodule is found incidentally on CXR


or CT

Chest pain as a symptom of lung cancer


caused by what

Pleural involvement or neoplastic invasion of the chest wall.

How to divide lung cancer

Small cell and non small cell


Of non small cell --> SCC, adenocarcinoma, and large cell carcinoma...
NSCLC 3-4x more commn than SCLC.

Lung cancer tha tis most likely to cavitate?

Squamous cell carcinoma. Usually a central / hilar lesion with local extension that may present with sx caused by bronchial
obstruction such as atelectasis and pneumonia.

What lung cancer is most likely to create


PTHrP

SCC

What lung cancers are generally more


peripheral

Adenocarcinoma
Large cell cancer
Adenocarcinoma mets early to CNS, bones, and adrenal glands.

SVC in the setting of lung cancer, most


likely wha ttype?

Large celll. This type tends to emt to the CNS and


mediastinum, causing SVC syndrome or hoarseness as
a consequence of laryngeal nerve paralysis.

Adenocarcinoma of the lung has a strong


association with what

Less with smoking


More with pulmoanry scars and fibrosis.

Small cell cancer behavior

Poorly differentiated cells of neuroendocrine origin, extremely aggressive but


also more likely to respond to chemotherapy/radiation but it unfortunately
relapses.
Cavitation never occurs... primary lesion is usually central. 80% have mets at
time of diagnosis...

SIADH
Ectopic ACTH
Eaton Lambert
Cushing
Peripheral neuropathy

Extrapulm manifestations of Small Cell

Prognosis of untreated SCLC

Contraindicatiosn to curative resection for


NSCLC

Predictor of seriousness of lung nodules

Grim --> survival is measured in weeks.


But 20-30% can be cured with radiotherapy or chemotherapy.

Extrathoracic
SVC syndrome
Vocal cord or phrenic nerve paralysis
Malignant pleural effusion
Cardiac tamponade
Tumor w/i 2cm of the carina
Mets to the other lung
Mets to supraclavicular lymph nodes or CI mediastinal node
Invovlement of main pulm artery

Size
0.2% for nodules <3mm
0.9% for 4-7 mm
18% for 8-20mm
50% for >20 mm
Greater than 99% of nodules <8 mm are benign
>1cm --> probably need a PET scan.

Lugn nodules appearance on imaging as a


predictor of malignancy

Popcorn or bullseye calcification --> benign


Absence of calcification --> increases
likelihood of malignancy.

SVC syndrome, why actually acutely life


threatening

Increased ICP
Fatal intracranial venous thrombosis

Pneumonia presents how

Cough, fever, pleuritic ches tpain, sputum


production, SOB, hypoxia, respiratory
distress.

Classic description of pneumococcal


pneumonia

Sudden onset of fever, cough with productive sputum, pleuritic chest


pain, rust colored sputum

Mycoplasma pneumonia may present how

Insidious onset, dry cough, prominent extrapulm


syptoms, headache, myalgias, sore throat, CXR that
appears much worse than auscultatory fundigns

Pathogens in HCAP

CURB-65

MRSA
Pseudomonas
Acinetobacter
MDR Enterobacteriaceae
Confusion
Urea >20
RR>30
BP<90
Age>65
0 - mortality rate of 1.5% 30 days
2 - 9.2% mortality --> admitted to hospital.

CXR findings in pneumonia

Every patient will have a visible pulmonary


infiltrate

Cavitation on CXR in the setting of


infection

Necrotizing infection:
Staph
TB
GN -->> Klebsiella

Cavitation is most likely in what lung


cancer

SCC

Sputum is minimally contaminated under


what circumstances

>25 PMNs
<10 epithelial cells per LPF
Diagnostic yield should be good.

What percent of pneumococcal


pneumonias become bacteremic

30-40%

Aspiration pneumonitis is what

Chemical injury to the lungs caused by aspiration of acidic gastric contents into
the lungs..... Usually sterile, but rather a chemical burn that causes a severe
inflammatory response... --> respriatory distress and a pulmonary infiltrate
4-6 hours appears, and typically resolves within 48 hours...

Aspiration pneumonia

Infectious proces causd by inhalation of oropharyngeal secretions


that are colonized by bacterial pathogens.

Recumbent patients aspirate into what

Posterior segments of upper lobes


Apical segments of lower lobes

How does legionella pneumonia typically


present

Adrenal insufficiency presentation

Fever, muscle aches, abdominal pain diarrhea


--> Nonproductive cough --> Severe pneumo.
Older adults.
Orthostatic hypotnesion
Intermittent chronic abdominal pain
Fatigue, weight loss
Hyponatremia, hyperkalemia, acidosis, hypoglycemia

Most common cause of adrenal


insufficiency in the developed world

Idiopathic autoimmune destruction

Steps in treating primary adrenal


insufficiency

IV Saline (volume depletion due to aldosteron


deficiency)
Glucose
Stress doses of corticosteroids

Primary adrenal insufficiency, causes

Addison disease --> adrenal failure to detruction or infiltration o fhte adrenal glands. Most
common cause worldwide is tuebrculous adrenalitis... Other causes can include chronic
granulomatous ifnection such as histo or coccidiomycosis), b/l adrenal hemorrhage (sepsis with
DIC), adrenal mets (commonly from lung, breast, or stomach cancers)
X-linked adrenoluekocydstrophy....
AIDS --> CMV or MAC

Secondary adrneal insufficiency

Lack of ACTH stimulation... Can be caused by autoimmune disorder or malginancy... Can be


autoimmune, malignant, infiltratative....
Most comon cause is CHRONIC EXOGENOUS ADMINISTRATION OF CORTICOSTEROIDS.
RAS system is usually able to function however.. So the patient is only deficient in cortisol.

Weakenss
Nausea
Vomiting
Abdominal pain

Acute adrenal insufficiency presentation

Fever
Hypotension
Tachcyardia,
FEVER

Lab findings in acute adrenal insufficiency

Hyponatremia
Hyperkalemia
Metabolic acidosis
Azotemia --> aldo deficiency
Hypoglycemia
Eosinophilia as a consequence of cortisol deficiency
May go into crisis when stressed by infection, trauma, or surgery... May look identical to septic shock, except for the
hypoglycemia and the hypotension that is corrected by steroids and not pressors.

CBC weirdness in acute adrenal


insufficiency

Cortisol levels, when high and when low

Eosinophilia --> Cortisol deficiency

High in morning
Low as day progresses
Morning plasma cortisol <5ug/dl in an acutely ill paitent is defintivie evidence
of adrenal insufficiency....
Random cortisol level>20 --> intact adrenal function.

ACTH stimulation test in which gland is


functioning just fine

How to test the entire hypothalamic


pituitary axis

Treatment of an Addisonian crisis

Increase in cortisol level of 7ug/dl


Maximal stimulated level >18 ug/dl
This is considered normal and indicates intact adrenal function.

Insulin-glucose tolerance test -->


Inducing hypoglycemia with insulin should result in
an increase in ACTH --> cortisol...
IV 5% glucose with normal saline
Adminsitration of corticosteroid therapy. HC is usually givne IV at doses of 100mg q6-8 h, or it can be given
by a bolus followed by continuous infusion...
At high doses, hydrocortisone provides both glucocorticoid and mineralocorticoid activity...
Draw a cortisol level before treatment to confirm the diagnosis. Causes of acute crisis should be
identified and treated, in particular, there should be a search for infection

Long term complications of glucocorticoid


excess

Diabetes
HTN
Obesity
Osteoporosis
Cataracts

Most common cause of rimary adrenal


insufficiency

Autoimmune adrenalitis

MRI to pick up ACTH producing tumors

Most are <5mm, and may not be seen on


MRI

Unconjugated bilirubin is bound to what

Serum reversibiliy and noncovalently


bound to albumin.

Hyperbili that results in visible jaundice

Serum bili >2-2.5 mg/dl

Unconjugated bilirubin definition

Gilbert syndrome

When conjugated bilirubin does not exceed 15% of total bilirubin. Almost
always caused by hemolysis or Gilbert syndrome... But in these syndromes
TBili <5mg/dl, and usually no sign of liver disease.

Benign condition caused by deficiency of hepatic enzymatic conjugation of


bilirubin, which results in intermittent unconjugated hyperbilirubinemia.
TBili<4, and is often precipitated by events such as stress, fasting, and febrile
illnesses.
NOT associated with liver dysfucntion, and requires no therapy

Conjugated hyperbili, usually what

First diagnostic test in patient with


cholestasis is what

Almsot always reflects either hepatocellular disease or biliary


obstruction.

US. Noninvasive and is very sensitive for detecting stones in the gallbladder as
well as intrahepatic or extrahepatic ductal dilatation.

Malignant causes of common bile duct and


pancreatic duct obstruction

Cholangiocarcinoma
Pancreatic cancer
Ampullary cancer

Serologic markers of PSC

None really

Complications of PSC vs PBC

PSC - Stricture, infection (cholangitis), cholangiocarcinoma

Courvoisiers

Nontender palpable gallbladder


Mild jaundice

PBC - cirrhosis

Causeunlikelyt to be gallstones

Target BP and LDL for DM-II

CAD equivalent
LDL<100
BP<130/80

Who should be screened for DM

Every 3 years beginning at age 45


Or earlier if BMI>25

Most senstiive test for DM

Glucose tolerance test to detect postprandial hyperglycemia

Special considerations of GLP-1 agonists

Injection
Nausea
Risk of pancreatitis

Specific test? Fasting plasma glucose

DPP-4 inhibitors, pecial considerations

Sulfonylureas in the elderly, concerns

No hypoglycemia or weight gain


Reduce dose in renal insufficiency

Increased renal insufficiency --> prolonged drug action -->


hypoglycemia.
Elderly, aim for more liberal A1c target like 8% instead of 7%

Glycemic goals for most diabetics

A1C<7
Preprandial glucose 70-130
Postprandial <180
BP <130/80
LDL <100

Possible precipitating factors for DKA

Infection
Pregnancy
Severe physiologic stressors --> MI

DKA triad

Anion gap metabolic acidosis


Hyperglycemia
Ketosis

What is the most important catabolic


hormone

Glucagon --> maintains normal glucose levels by stimulating hepatic gluconeogenesis and glycogenolysis.

Most common cause of death in DKA

Cardiovascular collapse

Lab values in DKA

Hyperglyecmia >250
Acidosis
Anion gap
Ketonemia
Serum potassium level is important.

Normal anion gap

Causes of high anion gap metabolic


acidosis

10-12

Lactic acidossis
Ketoacidosis (diabetic, alcoholic, starvation)
Toxins (ethylene glycol, methanol, salicylates)
Renal failure (acute or chronic, failur eto excrete acids)

SCr in DKA

Serum acetoacetate may cause false


elevation in SCr because of interference
with teh assay.

How often to measure glucose and


electrolytes in DKA

Glucose q1h
Electrolytes q3-5h

Best and first treatment for DKA

Insulin therapy in DKA

NS to correct circulatory collapse and volume deficit


1-2 L of NS over first hour...
Then the TBW deficit is correcte at a rate of 250-500 ml/hr depending on
state of hydration. Hydration should be gentler in CHF or ESRD because
patients can get easily fluid overloaded.
IV bolus of 0.1-0.15 U/kg
Followed by cont infusion 0.1 U/kg with hourly serum glucose determinations.
Decreased to 0.05 U/kg when BG level decreases to 250-300.... Keep in mind that glucose levels
fall more quickly as ketosis resolves...
<300 -- add 5-10% dextrose.... When bicarb is >18 --> anion gap is <12, the patient should feel
better, vital signs are stabilized...

Lab tests for ketones assess what

Acetoacetate and acetone


NOT beta-hydroxybutyrate
Insulin administration may result in the oxidation of betahydroxybutyrate to
acetoacetate, so ketone levels may incrase ith effective therapy. Thus
monitoring ketones is not particularly helpful.

In DKA, what shoul dbe given 30 mintue


sbeforestopping insulin infusion?

When should bicarb be given to DKA


patient

SC insulin, to avoid rebound acidosis.

Controversial, but <7 pH arterial...


Other indications such as cardiac instability or severe
hyperkalemia....
Bicarb can cause worsenign hypokalemia, paradoxical CNS
acidosis, and delay in ketone clearance.
Total body K, PO4, and Mg....

Electrolyte deficit in DKA

K should be given when K<5...


Once adequate urine output is establishe 20-40 mEq/L of fluid.
Cardiac monitoring is reocmmended in the presence of hypokalemia or hyperkalemia.
Phosphat egiven if <1.. or in hypophos patients with concomitant hypoxia, anemia, or cardiorespiratory compromise.
Careful monitoring of serum calcium level is necessary with phosphate administration.
Mg and Ca supplemneted as needed.

Complications of DKA

Cerebral edema
ARDS
Thromboembolism
Fluid overload
Acute gastric dilatation
Rare but serious complications all.

Labs in hyperosmolar nonketotic diabetic


coma

Glucose levels >1000mg/dl


Serum osm 320-370
Neuro symptoms rangign from confusion to seizures to coma...
Compared to DKA, much larger fluid deficit, therapy is primarily volume resus
with NS... Insulin given too but in lower doses than with DKA.

Alcoholic ketoacidosis

Chronic alcoholics who are malnourished and have depleted glycogen


stores --> often seen in the setting of binge drinking, which may shift
ratio of NADH to NAD inhbiting gluconeogenesis...
Develop anion gpa a s a result of ketoacidosis and lactic acidosis...

DKA vs alcoholic ketoacidosis presentation

Both present with symptoms of acidosis:


abdominal pain, nausea, vomiting, but with
low normal or slghtly elevated glucose levels...

Treatment for alcoholic ketoacidosis

NS
Glucose
Thiamine
Insulin is not necessary

Protuberant eyes
Systolic ejection murmur
Skin warm and dry
Resting tremor

PE findings assoc with hyperthyroid

Hyperthyroidism effects on cardiac system

Wide pulse pressure


Flow heart murmurs
Tachy
AFib - 10-20% of patients
Long standing thyroitoxicosis --> cardiomegaly and result in high output heart failure

Diarrhea in Hyperthyroidi?

Actually rare, just hyperdefecation

Exophthalmos in hyperthyroid?

Just distinctive for Graves disease.


Lid lag however, and wide eyed apperarance can be
hyperthyroid

Skin findings in hyperthyroid

Reproductive findings in hypothyroidism

"Apathetic hyperthyroid"

Warm, moist, velvety


Fine hair texture and alopecia
Sweating as a result of vasodilation and heat
dissipation
Impairs fertility in women and may cause
oligomenorrhea
Sperm count in men is reduced
Impotence and gynecomastia might be present.
Older atients may lack typical adrenergic features,
present instead with depression or apathy, weight
loss, AFib, worsening angina pectoris, or CHF

Findings in thyroid storm

Tachycardia (>140)
Fever (104-106)
Agitation
Delirium
Restlessness or psychosis
Vomiting and/or diarrhea

Settings of a thyroid storm

Long neglected severe hyperthryoidism to which a


complicating event (intercurrent illness, infection,
surgery, trauma or iodine load) is added...

Treatment of thyroid storm

IVF, antibiotics if needed, and speciifc treatment directed at hyperthyroidism...


Large doses of antithyroid meds
Iodine solution to blcok release of thyroid hormone
Propranolol to control symptoms by modulating adrenergic tone
Glucocorticoids to decrease T4--> T3 conersion

Most common cause of hyperthryoidism

Graes --> 80%, usually in women, esp b/w ages of 30-50 yo.

Graves disease signs

Goiter, thyroid bruit, hyperthyroidism, ophthalmopathy, and


dermopathy.. These features are variably present....
Ophthalmopathy can progress even after treatment of thyrotoicosis
because it is mediated by autoantibodies themselves.

Graves dermopathy

TPO antibodies in Graves or Hashimotos

Hyperpigmented orange peel texture papules, most commonly in


the shins --> Pretibial myxedema.

BOTH!
Graves has Thyroid Stimulating Immunoglobulin (TSI). Thyroid
uptake and scan --> reveal diffusely elevated iodine uptake in our
patient.

Medications for Graves

Propranolol --> symptom relief


Antithyroid drugs --> methmazole and
propylthiouracil (PTU).

Side effects of Methimazole and


Propylthiouracil

Rash,allergies
Arthritis
Hepatitis
Agranulocyutosios

Treatment of choice for Graves in the US

Radioactive iodine --> oral solution of 131I that is rapidly concentrated in thyroid tissue -->
radioablation, depending on dose, w/i 6-18 weeks...
CI in pregnant women, and women are advised to hold of pregnancy for 6-12 months after
treatment.
May exacerbate ophthalmopathy which glucocorticoids can help with.

Pregnant women with Graves best


treatment?

Propylthiouracil due to low placental transfer...


Surgery

Surgery for Graves?

Subtotal thyroidectomy for large gotiers with obstructive symptoms


(dyspnea, dysphagia)... Complications include laryngeal nerve
injury and hypoparathyroidism (due to removal or compromise of
vascular supply).

Other causes of thyrotoxicosis besides


Graves

Autonomous hyperfunctioning adenoma (Hot nodule or Plummer disease)


Thyroiditis or destruction of the thyroid gland resulgint in release of preformed hormone.
Subacute deQuervian thyroiditis is an inflammatory viral illness with thyroid pain and
tenderness.... Hyperthyroid phase lasts for several weeks or months, followed by recovery, but
some patients will develop hypothyroidism. Treatment with BB, usually sufficienct as is NSAIDs

Medications that can lead to


hyperthyroidism

Thyroid hormone
Amiodarone
Iodine load

Risk of thyroid cancer with hot vs cold


nodules

Hot almomst never


Cold 5-10% risk, so FNA surgical removal or US f/u is needed.

Who gets Toxic Multinodular Goiter

elderly and middle aged patients. Treatment consists


of radioactive iodine or surgery. Uptake is normal to
increased, and scan reveals irregular thyroid lobes
and a heterogeneous pattern.

Most common cause of thyrotoxicosis

Graves... No other diagnosis is likely if proptosis and a giter

Graves disease ever remit?

1/3 to 1/2 will become asymptomatic within 1-2 years on medical


therapy.

Criteria to distinguish transudative for


exudative pleural effusions

Light criteria, based on Protein and LDH


being high in the exudate.

How much fluid is needed to be visible on


a lateral decubitus film

50mL
>500 ml usually obscures the whole hemidiaphragm.

Bloody effusion >1/2 Hct of peripheral


blood vs <1/2

Indications for thorocentesis

>1/2 - hemothoraxi, or rupture of blood


vessel as seen in malignancy
<50% - cancer, PE, TB
Uneven pleural effusion or unilateral
Evidence of infection, e.g. productive cough, fever, pleurisy
Normal cardiac silhouette (no heart failure)
Alarm signs: weight loss, hemoptysis, hypoxia
Need to evaluate underlying lung parenchyma

How mcuh fluid can you remove via


thoracentesis before worrying about
reexpansion pulmonary edema

<=1500 ml

Effusions assoc with Pulmonary Emboli

Can acuse both exudative or transudative effusions,


may be bloody.
These are rarely large though.

If CHF is isolated, what side?

Rarely isolated, but most often right side.

Pleural LDH corresponds to what

Degree of pleural inflammation, and along with fluid protein, should always be sent in initial
evaluation.

How to treat multiloculated empyemeas

Fibrinolytic agents such as streptokinase or


urokinase
VATS

Most common causes of hemorrhagic


pleural effusions

Malignancy
PE w/infarction
TB

Familial hypercholesterol causes

Defective or absent LDL surface receptors --> subsequent inability


to metabolize LDL particles

LDL calculation

Total cholesterol - HDL - (Triglycerides/5)

What is considered premature CAD

Most common causes of secondary


dyslipidemia

Fibric acid derivatives side effects

Men<55
Women<65

Hypothyoidism
DM
Also:
Obstructive liver disease
Chronic renal failure/nephrotic syndrome
Med side effects: progestins, anabolic steroids, corticosteroids

Gallstones
Nausea
LFT increase

Monitor for dyspepsia, gallstones, myalgias (thus do not use with statins... can eb confusing).

What cholesterol lowering drug has little


evidence for dcreased CV risk

Ezetimibe

Thrombolytics time period

Within 3 hours.

How much stenosis before endarterctomy


is discussed

70%

Amaurosis fugax due to cholesterol emboli

Hollenhorst plaques whcih can be seen lodgedi nthe retinal artery.

Treatment of anaphylaxis

Epi
Steroids
Histamine blockers (H1 and H2)

Bee and wasp stings aka

Hymenoptera stings

Most common cause of drug-related


anaphylaxis
Food?

Beta lactam antibiotics


Peanuts

Positioning for anaphylaxis patient

Recumbent position, leg elevation

SJS vs TEN

Epidermal detachment>30% --> TEN

Serum sickness

Allertic raction tha toccurs 7-10 days after primary administration, or 2-4 days after secondary administration of foreng
serum or a drug.
Fever polyarthralgia, urtiaria, LADm and sometimes GN. Type III HSN reaction, caused by formatin of immune complexes
of IgG and the offending antigen
Usually self limiting, antihistamines, ASA, NSAIds

What is an "iodine allergy"

Radiologic contrast media --> result of hyperosmolar dye causing


degran of mast cells and basophils, rather than a true allergic
reaction. Can be prevented by diphenhydramine pretreatment, H2
blockers, and corticosteroids 12 h before procedure.

HSN syndorme associated with aromatic


anticonvulsants

Phenytoin, carbamazepine, phenobarb...severe idiosyncratic reaction not mediated by IgE that


includes rash, fever, and is often assoc wtih hepatitis, arthalgias, LAD, or heme abnormalities.

Pretreatment with what can greatly


decrease reaction to contrast dye

Diphenhydramine
H2 blockers
Corticosteroids

MMSE scores

Early
Intermediate 21-26
Late 10-20
Advanced <10

Polymyalgia rheumatica

Low grade fever


Diffuse muscle aches

Red flag HA

Cluster headache

Fundmanetla change
First severe and/or worst ha
Abrupt onset
Abnormal PE findings (general or neuro)
Neuro symptoms>1 hr
New HA <5y >50yo
New HA in patients with cancer, immunosuppression, pregnancy
HA assoc with alteration in or LOC
HA triggered by exertion, sex, VAlsalva

Male predominance
Precipitated by EtOH
Occurs with rhinorrhea, lacrimation

Brain tumor headache

Prostrating pounding HA that are assoc with


N/V
New "migraine" that is invariably unilateral

ACute angle closure glaucoma

Severe eye pain


N/V
Eye usually is painful and red
Pupil may be partially dilated

GCA how many patients go blind

Up to 20%

ACute cluster ha treatmetn

Oxygen
SQ sumatriptan

Age of temporal arteritis

Older than 50 years old

Osteopenia vs osteoporosis

T score b/1 -1 and -2.5 standard deviations below the


mean
OP <-2.5

T score vs Z score

T score - mean healty adult


Z score - BMD to persons of the same age
T score is more useful in predicting fracture risk

Recommendations for DEXA scan

Women>65 or those who have sustained a fracture before age 65

Das könnte Ihnen auch gefallen